Solved papers for RAJASTHAN ­ PET Rajasthan PET Solved Paper-2001

done Rajasthan PET Solved Paper-2001 Total Questions - 300

  • question_answer1) If anybody is rolling on surface, then its total energy will be

    A)
     \[\frac{1}{2}m{{v}^{2}}\,+\,\frac{1}{2}I{{\omega }^{2}}\]   

    B)
     \[\frac{1}{2}m{{v}^{2}}\,-\,\frac{G{{m}_{1}}{{m}_{2}}}{R}\]

    C)
     \[\frac{1}{2}m{{v}^{2}}\,-\,\frac{1}{2}\,m{{R}^{2}}{{\omega }^{2}}\]

    D)
     None of these

    View Answer play_arrow
  • question_answer2) A body of mass 5 kg is moving with velocity of 20 m/s. If the force of 200 N acts upon the body for 10 s, then finally the velocity of body will be

    A)
     150 m/s          

    B)
     220 m/s

    C)
     200 m/s

    D)
     240 m/s

    View Answer play_arrow
  • question_answer3) What will be the moment of inertia of a ring about tangential axis in its plane?

    A)
     \[M{{R}^{2}}\]        

    B)
     \[2M{{R}^{2}}\]

    C)
     \[\frac{3}{2}M{{R}^{2}}\]

    D)
     None of these

    View Answer play_arrow
  • question_answer4) The escape velocity of an object on a planet whose gravity value is 2 times as on earth and whose radius is 2 times that of earth will be

    A)
     \[2{{V}_{e}}\]           

    B)
     \[3{{V}_{e}}\]

    C)
     \[4{{V}_{e}}\]             

    D)
     None of these

    View Answer play_arrow
  • question_answer5) Dimensions of universal gravitational constant will be

    A)
     \[[M{{L}^{3}}{{T}^{-2}}]\]        

    B)
     \[[M{{L}^{-1}}{{L}^{3}}{{T}^{-2}}]\]

    C)
     \[[{{M}^{-1}}{{L}^{2}}{{T}^{2}}]\]

    D)
     None of these

    View Answer play_arrow
  • question_answer6) If y = 2.4\[\eta \]the poisson ratio is

    A)
     \[-1\]                

    B)
     0.2

    C)
     0.1                

    D)
    \[-0.25\]

    View Answer play_arrow
  • question_answer7) Which of the following quantity are not same?

    A)
     Planck's constant and angular momentum

    B)
     Work and energy

    C)
     Pressure and Young's modulus

    D)
     Torque of force and moment of inertia

    View Answer play_arrow
  • question_answer8) The ratio of traveled distances by freely falling body in first, second and third seconds will be

    A)
     5 : 3 : 1           

    B)
     1 : 4 : 9

    C)
     1 : 3 : 5           

    D)
     9 : 4 : 1

    View Answer play_arrow
  • question_answer9) What will be the value of gravitational acceleration at height h from surface of earth? If h>>R, where R is radius of earth and gravitational acceleration at surface of earth is g.

    A)
     \[\frac{g}{{{\left( 1\,+\,\frac{h}{R} \right)}^{2}}}\]

    B)
     \[g\left( 1-\frac{2h}{g} \right)\]

    C)
     \[\frac{g}{{{\left( 1-\frac{h}{R} \right)}^{2}}}\]

    D)
     \[g{{\left( 1-\frac{h}{R} \right)}^{{}}}\]

    View Answer play_arrow
  • question_answer10) Two trains of length 50 m are crossing each other. If velocities in opposite directions are 10 m/s and 15 m/s, then time taken to cross each other will be

    A)
     2s               

    B)
     4s

    C)
     6s               

    D)
     8s

    View Answer play_arrow
  • question_answer11) Two springs of force constants k1 = 500 N/m and \[{{k}_{2}}=3000\text{ }N/m\]stretched by same force. Therefore ratio of their potential energies will be

    A)
     2 : 1            

    B)
     1 : 2

    C)
     4 : 1            

    D)
     1 : 4

    View Answer play_arrow
  • question_answer12) If potential and current are\[V=(100\pm 0.5)\]volt and\[I=(10\pm 0.2)\text{ }A\]respectively, then value of resistance will be

    A)
     (10 ± 0.7)\[\Omega \]       

    B)
     (5 ± 2)\[\Omega \]

    C)
     (0.1 ± 2)\[\Omega \]        

    D)
     None of these

    View Answer play_arrow
  • question_answer13) The range of projectile is four times of its maximum height, then angle of projection will be

    A)
     \[60{}^\circ \]               

    B)
     \[45{}^\circ \]

    C)
     \[30{}^\circ \]               

    D)
     \[75{}^\circ \]

    View Answer play_arrow
  • question_answer14) What will be the orbital velocity of planet revolves near earth (where R is radius of earth and g is gravitational acceleration)?

    A)
     \[\sqrt{Rg}\]             

    B)
     \[\sqrt{2Rg}\]

    C)
     \[2\sqrt{Rg}\]           

    D)
     \[\sqrt{3Rg}\]

    View Answer play_arrow
  • question_answer15) If bullets of mass m strikes on a wall at the rate of 27 per second, then what will be the force applied on the wall?

    A)
     mnv             

    B)
     4mnv

    C)
     2mnv

    D)
     \[\frac{mnv}{2}\]

    View Answer play_arrow
  • question_answer16) If the length of simple pendulum increases 1%, then change in time period will be

    A)
     0.5%            

    B)
     1%

    C)
     2%             

    D)
     0.2%

    View Answer play_arrow
  • question_answer17) Two bodies of mass 1 kg are placed at a distance of 1 m have each other. The force between two bodies will be

    A)
     equal to G  

    B)
     equal to gravitation

    C)
     equal to G / 2     

    D)
     None of these

    View Answer play_arrow
  • question_answer18) As per given figure to complete the circular loop what should be its radius, if initial height is 5m?

    A)
     4 m                    

    B)
     3 m           

    C)
     2.5 m                    

    D)
     2 m         

    View Answer play_arrow
  • question_answer19) The particle moves according to given velocity-time graph. The ratio of travelling distance in last 2s and 7s will be

    A)
     \[\frac{1}{4}\]

    B)
     \[\frac{1}{2}\]

    C)
     \[\frac{1}{8}\]

    D)
     \[\frac{1}{6}\]

    View Answer play_arrow
  • question_answer20) On playing sitar the nature of wave on the reaching the observer will be

    A)
     transverse and progressive

    B)
     longitudinal and progressive

    C)
     transverse and stationary

    D)
     longitudinal and stationary

    View Answer play_arrow
  • question_answer21) The gravitation of earth is 10 m/s. If the mass of earth is 80 times, the mass of moon and radius of earth is 4 times the radius of moon, then the gravitation at moon will be

    A)
     \[\text{1 }m/{{s}^{2}}\]          

    B)
     \[2\text{ }m/{{s}^{2}}\]

    C)
     \[\text{3 }m/{{s}^{2}}\]           

    D)
     None of these

    View Answer play_arrow
  • question_answer22) The maximum acceleration in SHM will be

    A)
     at amplitude

    B)
     at equilibrium position

    C)
     at any point

    D)
     None of the above

    View Answer play_arrow
  • question_answer23) The value of \[{{\varepsilon }_{0}}\] will be

    A)
     \[8.85\,\times \,{{10}^{-12}}{{C}^{2}}/N-{{m}^{2}}\]

    B)
     \[8.85\,\times \,{{10}^{-12}}\,Fermi\]

    C)
     \[9\,\times \,{{10}^{-13}}\,{{C}^{2}}/N-{{m}^{2}}\]

    D)
     \[9\,\times \,{{10}^{-13}}\,Fermi\]

    View Answer play_arrow
  • question_answer24) Rain water falls horizontally downwards with velocity v. When the velocity of air in horizontal direction is u, then water is store at the rate R m3/s. When the velocity of air becomes 2u, the rate of store of water will be

    A)
     R                

    B)
     R/2

    C)
     2R             

    D)
     R\[\frac{\sqrt{4{{u}^{2}}\,+\,{{v}^{2}}}}{\sqrt{{{u}^{2}}+{{v}^{2}}}}\]

    View Answer play_arrow
  • question_answer25) In Carnot cycle, the temperature of source is 500 K. If engine lakes in 300 cal of heat from a source and gives 150 cal of heat to a sink. The temperature of sink will be

    A)
     500 K           

    B)
     250 K

    C)
     750 K           

    D)
     125 K

    View Answer play_arrow
  • question_answer26) The average kinetic energy per molecule of the gas is

    A)
     \[\frac{1}{2}kT\]           

    B)
     \[\frac{3}{2}kT\]

    C)
     \[\frac{1}{2}RT\]           

    D)
     \[\frac{3}{2}RT\]

    View Answer play_arrow
  • question_answer27) If the temperature of gas increases from\[27{}^\circ C\]to \[927{}^\circ C,\]the KE will be

    A)
     double          

    B)
     half

    C)
     one-fourth       

    D)
     four times

    View Answer play_arrow
  • question_answer28) Choose the correct relation. (where p = pressure, E = kinetic energy of gas)

    A)
     \[p=\frac{3}{2}E\]

    B)
     \[p=\frac{2}{3}E\]

    C)
     \[p=\frac{1}{2}E\]

    D)
     \[p=\,2E\]

    View Answer play_arrow
  • question_answer29) For a perfectly black body, its absorption coefficient will be

    A)
     1                

    B)
     0

    C)
     between 0 and 1  

    D)
     None of these

    View Answer play_arrow
  • question_answer30) The heat given to diatomic gas at constant pressure, the how many quantity will be change in internal energy?

    A)
     5/7              

    B)
     3/5

    C)
     2/5               

    D)
     None of these

    View Answer play_arrow
  • question_answer31) The relation between velocities of\[{{O}_{2}}\]and\[{{H}_{2}}\]

    A)
    \[v{{H}_{2}}=4{{v}_{O2}}\]       

    B)
     \[{{v}_{O2}}=4{{V}_{H2}}\]

    C)
     \[{{v}_{O2}}=2{{v}_{H2}}\]       

    D)
     \[{{v}_{H2}}=2{{v}_{O2}}\]

    View Answer play_arrow
  • question_answer32) The quality of metal of cooking vessel

    A)
     less specific heat and more conduction

    B)
     more specific heat and less conduction

    C)
     more specific heat and more conduction

    D)
     less specific heat and less conduction

    View Answer play_arrow
  • question_answer33) Volume of gas becomes four times if

    A)
     temperature becomes four times at constant pressure

    B)
     temperature becomes one fourth at constant pressure

    C)
     temperature becomes two times at constant pressure

    D)
     temperature becomes half at constant pressure

    View Answer play_arrow
  • question_answer34) A source of frequency 150 Hz is moving in the direction of a person with a velocity of 110 m/s. The frequency heard by the person will be (speed of sound medium = 330 m/s)

    A)
     225 Hz          

    B)
     250 Hz

    C)
     495 Hz          

    D)
     None of these

    View Answer play_arrow
  • question_answer35) To increase the frequency from 100 Hz to 400 Hz, the tension of string will be

    A)
     4 times          

    B)
     16 times

    C)
     20 times         

    D)
     None of these

    View Answer play_arrow
  • question_answer36) A big ball of mass M, moving with velocity u strikes a small ball of mass m, which is at rest. Finally small ball obtains velocity u and big ball v. Then what is the value of v?

    A)
     \[\frac{M-m}{M}u\]

    B)
     \[\frac{m}{M+m}u\]

    C)
     \[\frac{2m}{M+m}u\]

    D)
     \[\frac{M}{M+m}u\]

    View Answer play_arrow
  • question_answer37) Twenty seven drops of water of the same size are equally potential V. They are then united to form a bigger drop. By what factor will the electrical potential changes?

    A)
     9V

    B)
     3V

    C)
     27V            

    D)
     \[\frac{1}{3}V\]

    View Answer play_arrow
  • question_answer38) The power factor of L-C- R circuit at resonance is

    A)
     zero               

    B)
     1

    C)
     0.5                

    D)
     0.766

    View Answer play_arrow
  • question_answer39) An alternating current of frequency / is flowing in a circuit containing a resistance R and choke L in series. The impedance of this circuit is

    A)
     \[\sqrt{{{R}^{2}}+\,{{(2\pi fL)}^{2}}}\]      

    B)
     \[\sqrt{{{R}^{2}}+\,{{(2\pi {{f}^{2}})}^{2}}}\]

    C)
     \[\sqrt{{{R}^{2}}+\,L\pi {{f}^{2}}}\]         

    D)
      \[{{R}^{2}}-\,{{(2\pi {{f}^{2}})}^{2}}\]

    View Answer play_arrow
  • question_answer40) V rms = 220 V, then the peak value of voltage is

    A)
     311 V           

    B)
     281 V

    C)
     200 V           

    D)
     181 V

    View Answer play_arrow
  • question_answer41) The magnetic moment of coil is M and area is A. Then value of current will be

    A)
     \[\frac{2M}{A}\]

    B)
     \[\frac{M}{A}\]

    C)
     \[\frac{A}{M}\]

    D)
     \[\frac{M}{2A}\]

    View Answer play_arrow
  • question_answer42) In the circuit shown below, the current of 2A is flowing, the impedance of circuit will be

    A)
     170 \[\Omega \]          

    B)
     70 \[\Omega \]

    C)
     130 \[\Omega \]           

    D)
     120 \[\Omega \]

    View Answer play_arrow
  • question_answer43) In AC circuit\[E={{E}_{0}}sin\]at and\[I={{I}_{0}}sin\]\[\left( \omega t-\frac{\pi }{2} \right)\]then power dissipated will be

    A)
     zero            

    B)
     \[\frac{{{E}_{0}}{{I}_{0}}}{2}\]

    C)
     \[{{E}_{rms}}{{I}_{rms}}\]         

    D)
     None of these

    View Answer play_arrow
  • question_answer44) If\[{{C}_{air}}=10\]\[\mu \]F, then effective capacity will be

    A)
     30\[\mu \]F           

    B)
     15\[\mu \]F

    C)
     3 \[\mu \]F            

    D)
     10 \[\mu \]F

    View Answer play_arrow
  • question_answer45) A voltmeter has a resistance of G ohm and range V volt. The value of resistance used in series to convert it into a voltmeter of range nV volt is

    A)
     nG              

    B)
     (n-1)G

    C)
     \[\frac{G}{n}\]       

    D)
     \[\frac{G}{(n-1)}\]

    View Answer play_arrow
  • question_answer46) The magnetic moment of current loop is\[2.1\times {{10}^{-25}}\]\[A-{{m}^{2}}\]. The magnetic field at distance 1 \[\overset{o}{\mathop{\text{A}}}\,\]from centre on axis of loop will be

    A)
     \[4.2\times {{10}^{2}}Wb/{{m}^{2}}\]

    B)
     \[4.2\times {{10}^{-3}}Wb/{{m}^{2}}\]

    C)
     \[4.2\times {{10}^{-4~}}Wb/{{m}^{2}}\]

    D)
     \[4.2\times {{10}^{-5}}Wb/{{m}^{2}}\]

    View Answer play_arrow
  • question_answer47) The intensity of electric field at a point on the axis of an electric dipole depends on the distance r from the centre of the dipole as

    A)
     \[E\propto \frac{1}{{{r}^{2}}}\]                

    B)
     \[E\,\propto \frac{1}{{{r}^{3}}}\]

    C)
     \[E\propto \,{{r}^{2}}\]           

    D)
     \[E\propto \,\frac{1}{{{r}^{4}}}\]

    View Answer play_arrow
  • question_answer48) How many electrons are in 1 coulomb charge

    A)
     \[6.25\times {{10}^{18}}\]       

    B)
     \[1.6\times {{10}^{19}}\]

    C)
     100             

    D)
     \[6.25\times {{10}^{12}}\]

    View Answer play_arrow
  • question_answer49) When charge q enters perpendicular in magnetic field B, then frequency will be

    A)
     \[\frac{2\pi m}{qB}\]

    B)
     \[\frac{qB}{2\pi m}\]

    C)
     \[\frac{qB}{\pi m}\]             

    D)
     \[\frac{\pi m}{qB}\]

    View Answer play_arrow
  • question_answer50) The resistance of ammeter is 99ft. The current of ammeter is 10% of main current, then the resistance of shunt will be

    A)
     11\[\Omega \]            

    B)
     22\[\Omega \]

    C)
     99\[\Omega \]          

    D)
     33\[\Omega \]

    View Answer play_arrow
  • question_answer51) \[E=2\text{ }V/m\]at distance 60 cm from the charge, then charge will be

    A)
     \[8\times {{10}^{-11}}C\]      

    B)
     \[8\times {{10}^{11}}C\]

    C)
     \[4\times {{10}^{11}}C\]       

    D)
     \[4\times {{10}^{-11}}C\]

    View Answer play_arrow
  • question_answer52) AB is a wire of uniform resistance. The galvanometer G shows no current when the length AB=20 cm and BC =80 cm. The resistance  R is equal to

    A)
     20\[\Omega \]

    B)
     80\[\Omega \]                 

    C)
     10\[\Omega \]         

    D)
     40\[\Omega \]

    View Answer play_arrow
  • question_answer53) The potential difference on capacitor of 4.5 \[\mu \]F

    A)
     8 V              

    B)
     4 V                 

    C)
     2 V                             

    D)
     6 V             

    View Answer play_arrow
  • question_answer54) If in a transformer, the Lechlanche cell is connected across the primary, then in the secondary coil

    A)
     emf will be induced

    B)
     any emf will not be induced

    C)
     emf will be increase

    D)
     emf will be decrease

    View Answer play_arrow
  • question_answer55) The unit of inductance

    A)
     \[\frac{volt\,\times \,\sec ond}{ampere}\]

    B)
      \[\frac{volt\,\times \,ampere}{\sec ond}\]

    C)
     \[\frac{volt}{ampere}\]

    D)
     \[\frac{volt\,\times \,{{(ampere)}^{2}}}{\sec ond}\]

    View Answer play_arrow
  • question_answer56) Where does the intensity of electric field become zero in hollow sphere?

    A)
     At external points

    B)
     At outer surface

    C)
     At internal points

    D)
     At inside and outside

    View Answer play_arrow
  • question_answer57) In Young's double slit experiment, the screen is kept at a distance 2m from source. The distance between light wavelength of 6000\[\overset{o}{\mathop{\text{A}}}\,\]and phase mode source is 4 mm. The band width will be

    A)
     0.2 m           

    B)
     2 cm

    C)
     20 cm           

    D)
     0.03 cm

    View Answer play_arrow
  • question_answer58) The mean distance between the atoms of metal is \[3\times {{10}^{-10}}m\]and interatomic force constant for metal is\[3.6\times {{10}^{-9}}N/A,\]then Young's modulus of metal is

    A)
     \[1.2\times {{10}^{11}}N/{{m}^{2}}\]  

    B)
     \[4.2\times {{10}^{11}}N/{{m}^{2}}\]

    C)
     \[10.8\times {{10}^{-19}}N/{{m}^{2}}\]

    D)
     \[2.4\times {{10}^{10}}N/{{m}^{2}}\]

    View Answer play_arrow
  • question_answer59) If light polarized by the reflection then the angle between reflected wave and refracted wave will be

    A)
     \[\pi \]                 

    B)
     \[\pi /2\]

    C)
     \[2\pi \]                

    D)
     \[\pi /4\]

    View Answer play_arrow
  • question_answer60) Condition of diffraction is

    A)
     \[\frac{a}{\lambda }=1\]          

    B)
     \[\frac{a}{\lambda }>>1\]

    C)
     \[\frac{a}{\lambda }<<1\]             

    D)
     None of these

    View Answer play_arrow
  • question_answer61) If photoelectric  effect experiment, the intensity of incident ray increases, then the correct statement is

    A)
     the kinetic energy of emitted electron will be maximum

    B)
     work function will be unchanged

    C)
     stopping potential will be increases

    D)
     the kinetic energy of emitted electron will be become less

    View Answer play_arrow
  • question_answer62) During the \[{{\beta }^{-}}\] decay

    A)
     neutron, converts into proton

    B)
     proton converts into neutron

    C)
     neutron-proton ratio increases

    D)
     None of the above

    View Answer play_arrow
  • question_answer63) In depletion layer, there are

    A)
     immobile ions

    B)
     free charge carries

    C)
     charge carrier and immobile ions

    D)
     None of the above

    View Answer play_arrow
  • question_answer64) The relation among the forbidden energy band gap in conductors, semiconductors and insulators is

    A)
     \[\Delta E{{g}_{c}}\]>\[\Delta E{{g}_{s}}\]>\[\Delta E{{g}_{i}}\]

    B)
     \[\Delta E{{g}_{i}}\]>\[\Delta E{{g}_{s}}\]>\[\Delta E{{g}_{c}}\]

    C)
     \[\Delta E{{g}_{c}}\]>\[\Delta E{{g}_{c}}\]>\[\Delta E{{g}_{s}}\]

    D)
     \[\Delta E{{g}_{s}}\]>\[\Delta E{{g}_{c}}\]>\[\Delta E{{g}_{s}}\]

    View Answer play_arrow
  • question_answer65) By increasing the temperature of semi- conductors, its resistance

    A)
     increase         

    B)
     decrease

    C)
     remain constant  

    D)
     None of these

    View Answer play_arrow
  • question_answer66) The value of current in the circuit

    A)
     zero               

    B)
     1 A

    C)
     0.1 A              

    D)
     0.2 A

    View Answer play_arrow
  • question_answer67) Good nuclear fuel is

    A)
     \[{{U}^{238}}\]             

    B)
     \[P{{u}^{238}}\]

    C)
     \[T{{h}^{234}}\]            

    D)
     \[{{U}^{235}}\]

    View Answer play_arrow
  • question_answer68) The binding energy of K-shell of electron is\[-40000\]eV. Applied the potential of 60000 V in Coolidge tube, X-rays will be

    A)
     continuous X-ray

    B)
     white X-rays

    C)
     continuous and all characteristics X-rays

    D)
     None of the above

    View Answer play_arrow
  • question_answer69) The typical ionization energy of a donor in silicon is

    A)
     10.0 eV         

    B)
     1.0 eV

    C)
     0.1 eV           

    D)
     0.001 Ev

    View Answer play_arrow
  • question_answer70) If the threshold frequency of sodium metal is 6800 A, then work function will be

    A)
     1.8 eV           

    B)
     2.5 eV

    C)
     2.1 eV           

    D)
     1.4 eV

    View Answer play_arrow
  • question_answer71) If elements with quantum number n > 4 are not in the nature, the number of possible elements is

    A)
     60              

    B)
     32

    C)
     4               

    D)
     64

    View Answer play_arrow
  • question_answer72) If capillary experiment is performed in vacuum, then for a liquid there

    A)
     it will rise        

    B)
     will remain same

    C)
     it will fall        

    D)
     rise to the top

    View Answer play_arrow
  • question_answer73) The reason of Avalanche breakdown is

    A)
     collision of minority charge carriers

    B)
     increasing of depletion layer

    C)
     decreasing of depletion layer

    D)
     None of the above

    View Answer play_arrow
  • question_answer74) Which of the following relations are correct?

    A)
     \[E=\,\frac{hc}{\lambda }\]

    B)
     \[E=\,\frac{1}{2}m{{u}^{2}}\]

    C)
     \[P=\,\frac{E}{2v}\]

    D)
     \[E=\,\frac{1}{2}m{{c}^{2}}\]

    View Answer play_arrow
  • question_answer75) Ionic potential for second electron of helium is

    A)
     13.6 eV          

    B)
     27.2 eV

    C)
     54.4 eV          

    D)
     100 eV

    View Answer play_arrow
  • question_answer76) Which of the following relations are correct for characteristics X-rays?

    A)
     \[\lambda \,\alpha \,\frac{1}{{{Z}^{2}}}\]

    B)
     \[\lambda \,\alpha \,{{Z}^{2}}\]

    C)
     \[\lambda \times Z\]

    D)
     \[1\,\alpha \frac{1}{V}\]

    View Answer play_arrow
  • question_answer77) The beam of 1 \[\mu \]A of proton whose cross-section area is\[0.5\text{ }m{{m}^{2}}\]and velocity is\[3\times {{10}^{4}}m/s,\]then charge density of beam

    A)
     \[6.6\,\times \,{{10}^{-4}}C/{{m}^{3}}\]

    B)
     \[6.6\,\times \,{{10}^{-5}}C/{{m}^{3}}\]

    C)
     \[6.6\,\times \,{{10}^{-6}}C/{{m}^{3}}\]

    D)
     None of these

    View Answer play_arrow
  • question_answer78) The correct relation for elastic potential energy is

    A)
     energy density = \[\frac{1}{2}\]\[\times \]strain\[\times \]stress

    B)
     energy density =\[{{(strain)}^{2}}\]\[\times \] volume

    C)
     energy density = strain \[\times \] volume

    D)
     energy density = \[{{(strain)}^{2}}\]\[\times \] volume

    View Answer play_arrow
  • question_answer79) The ratio of the longest to shortest wavelengths in Brackett series of hydrogen spectrum is

    A)
     \[\frac{25}{9}\]              

    B)
     \[\frac{17}{6}\]

    C)
     \[\frac{9}{5}\]

    D)
     \[\frac{4}{3}\]

    View Answer play_arrow
  • question_answer80) An Indian rubber cord L metre long and area of cross-section A metre2 is suspended vertically. Density of rubber is\[D\text{ }kg/{{m}^{2}}\]and Young's modulus of rubber is\[E\text{ }n/{{m}^{2}}\]. If the wire extends by 1 m under its own height; then extension in length / is

    A)
     \[{{L}^{2}}Dg/E\]

    B)
     \[{{L}^{2}}Dg/2E\]

    C)
     \[{{L}^{2}}Dg/4E\]

    D)
     \[L\]

    View Answer play_arrow
  • question_answer81) The nature of electromagnetic waves is

    A)
     transverse       

    B)
     logitudinal

    C)
     Both [a] and [b]   

    D)
     None of these

    View Answer play_arrow
  • question_answer82) Radius of a soap bubble is increased from R to 2R, work done is this process in terms of surface tension is

    A)
     \[24\pi {{R}^{2}}T\]         

    B)
     \[12T\pi {{r}^{2}}\]

    C)
     \[6T\pi {{r}^{2}}\]

    D)
     \[4T\pi {{r}^{2}}\]

    View Answer play_arrow
  • question_answer83) Light is incident normally on diffraction grating through which the first order, diffraction is seen\[32{}^\circ \]. The second order diffraction will be seen at

    A)
     \[48{}^\circ \]

    B)
     \[64{}^\circ \]

    C)
     \[80{}^\circ \]

    D)
     There is no second order diffraction in this case

    View Answer play_arrow
  • question_answer84) If liquid level fall in a capillary, then radius of curvature in capillary tube will

    A)
     increase          

    B)
     decrease

    C)
     unchanged      

    D)
     None of these

    View Answer play_arrow
  • question_answer85) The relative velocity of two consecutive layer is 8 cm/s. If the perpendicular distance between the layers is 0.1 m, then the velocity gradient will be

    A)
     \[50\text{ }{{s}^{-1}}\]            

    B)
     \[\text{60 }{{s}^{-1}}\]

    C)
     \[\text{75 }{{s}^{-1}}\]            

    D)
     \[\text{80 }{{s}^{-1}}\]

    View Answer play_arrow
  • question_answer86) The pressure below in surface area of water in the capillary tube will be

    A)
     equal to atmospheric pressure

    B)
     equal to above pressure

    C)
     less than above pressure

    D)
     more than above pressure

    View Answer play_arrow
  • question_answer87) The ratio of intensities of bright and dark fringes are given by 4:1. The ratio of the amplitudes of the two waves is

    A)
     1 : 1             

    B)
     3 : 1

    C)
     1 : 3             

    D)
     1 : 9

    View Answer play_arrow
  • question_answer88) An object is at a temperature of\[400{}^\circ C\]. At what temperature would it radiate energy twice as fast? The temperature of the surroundings may be assumed to be negligible

    A)
     \[200{}^\circ C\]           

    B)
     200 K

    C)
     \[800{}^\circ C\]           

    D)
     800 K

    View Answer play_arrow
  • question_answer89) tress to strain ratio is equivalent to

    A)
     Modulus of elasticity

    B)
     Poisson's ratio

    C)
     Reynold number

    D)
     Fund number

    View Answer play_arrow
  • question_answer90) In an isothermal reversible expansion, if the volume of 96 g of oxygen at\[27{}^\circ C\]is increased from 70 litre to 140 litre, then the work done by the gas will be

    A)
     \[300\text{ }R\text{ }lo{{g}_{10}}2\]    

    B)
     \[81\text{ }R\text{ }lo{{g}_{e}}2\]

    C)
     \[900\text{ }R\text{ }lo{{g}_{10}}2\]     

    D)
     \[2.3\times 900\text{ }R\text{ }lo{{g}_{10}}2\]

    View Answer play_arrow
  • question_answer91) To get the maximum current from a parallel combination of n identical cells each of internal resistance r in an external resistance R when

    A)
     R>>r              

    B)
     R<<r

    C)
     R = r            

    D)
     None of these

    View Answer play_arrow
  • question_answer92) A mass M is suspended by two springs of force constants\[{{k}_{1}}\]and\[{{k}_{2}}\]respectively as shown in the diagram. The total elongation (stretch) of the two springs is

    A)
     \[\left( \frac{{{k}_{1}}+{{k}_{2}}}{{{k}_{1}}{{k}_{2}}} \right)\]

    B)
     \[\frac{2({{k}_{1}}+{{k}_{2}})}{{{k}_{1}}{{k}_{2}}}mg\]

    C)
     \[\frac{2({{k}_{1}}+{{k}_{2}})}{{{k}_{1}}{{k}_{2}}}mg\]              

    D)
     None of the above          

    View Answer play_arrow
  • question_answer93) A potentiometer has uniform potential gradient. The specific resistance of the material of the potentiometer wire is \[40\times {{10}^{-8}}m\] and current passing through it is 15 A, cross-section area of the wire is\[18\times {{10}^{-6}}{{m}^{2}}\]. The potential gradient along the potentiometer wire is

    A)
     25 V/m          

    B)
     2.5 V/cm

    C)
     2.5 V/m          

    D)
     0.33 V/m

    View Answer play_arrow
  • question_answer94) Work done in splitting a drop of water of 1 mm radius into \[{{10}^{6}}\]droplets is (surface tension of water\[=72\times {{10}^{-3}}J/{{m}^{2}}\])

    A)
     \[89.5\times {{10}^{-5}}J\]      

    B)
     0.895 J

    C)
     \[895\times {{10}^{-5}}J\]      

    D)
     None of these

    View Answer play_arrow
  • question_answer95) The size of magnet to generate the radial magnetic field in moving coil galvanometer will be

    A)
     concave         

    B)
     horse shoe magnet

    C)
     convex          

    D)
     None of these

    View Answer play_arrow
  • question_answer96) Transverse elastic wave can passes

    A)
     in gas and solid

    B)
     in solid but not in gas

    C)
     in gas but not in solid

    D)
     None of the above

    View Answer play_arrow
  • question_answer97) The equivalent resistance between the points A and B is

    A)
     1\[\Omega \]       

    B)
     2\[\Omega \]

    C)
     3\[\Omega \]

    D)
     4\[\Omega \]

    View Answer play_arrow
  • question_answer98) A potentiometer has uniform potential gradient across it. Two cells connected in series (i) to support each other and (ii) to oppose each other are balanced over 6 m and 2 m respectively on the potentiometer wire. The emf's of the cells are in the ratio of

    A)
    1 : 2   

    B)
    1 : 1   

    C)
    3 : 1   

    D)
    2 : 1

    View Answer play_arrow
  • question_answer99) A Carnot engine working between 300 K and 600 K has work output of 800 j per cycle. What is amount of heat energy supplied to the engine from source, per cycle?

    A)
     1800 J

    B)
     1000 J

    C)
     2000 J

    D)
     1600 J

    View Answer play_arrow
  • question_answer100) The ideal gas expands in such a manner that its pressure and volume can be related by equation\[p{{V}^{2}}=\]constant. During this process, the gas is

    A)
     heated

    B)
     cooled

    C)
     neither heated nor cooled

    D)
     first heated and then cooled

    View Answer play_arrow
  • question_answer101) Structure of\[{{C}_{2}}{{H}_{2}}\] is

    A)
     linear

    B)
     trigonal planar

    C)
     pyramidal       

    D)
     None of these

    View Answer play_arrow
  • question_answer102) In \[CsCl\]crystal, if\[C{{s}^{+}}\] ion is surrounded from \[8C{{l}^{-}}\]ions and \[C{{l}^{-}}\]ion from\[C{{s}^{+}}\] ion then coordination number will be

    A)
     4               

    B)
     6

    C)
     8               

    D)
     12

    View Answer play_arrow
  • question_answer103) Lattice energy is

    A)
     energy used in the formation of 1 mole of solid crystal

    B)
     energy released in the formation of 1 mole of solid crystal

    C)
     energy released in the formation of 1 g of solid crystal

    D)
     None of the above

    View Answer play_arrow
  • question_answer104) If the value of \[{{K}_{a}}\] for acid is (I) \[1.36\times {{10}^{-5}}\] (II) \[125\times {{10}^{-5}}\] (III) \[260\times {{10}^{-5}}\] (IV) \[67\times {{10}^{-5}}\] then the strongest acid will be

    A)
     I                 

    B)
     II

    C)
     III               

    D)
     IV

    View Answer play_arrow
  • question_answer105) A solution obtained on mixing 0.1 N, 50 mL \[HCl\]and 0.1 N, 50 mL\[NaOH\]. The value of pH of the solution will be

    A)
     4               

    B)
     6

    C)
     8               

    D)
     7

    View Answer play_arrow
  • question_answer106) The oxidation number of Cr in\[{{K}_{2}}C{{r}_{2}}{{O}_{7}}\]is

    A)
     6                

    B)
     4

    C)
     7                

    D)
     3

    View Answer play_arrow
  • question_answer107) Correct order of bond length is

    A)
     \[HC\equiv CH>{{H}_{2}}C=C{{H}_{2}}>{{H}_{3}}C-C{{H}_{3}}\]

    B)
     \[{{H}_{2}}C=C{{H}_{2}}>HC\equiv CH>{{H}_{3}}C-C{{H}_{3}}\]

    C)
     \[C{{H}_{3}}-C{{H}_{3}}>C{{H}_{2}}=C{{H}_{2}}>HC\equiv CH\]

    D)
     \[C{{H}_{3}}-C{{H}_{3}}>HC\equiv CH>{{H}_{2}}C=C{{H}_{2}}\]

    View Answer play_arrow
  • question_answer108) \[A+BC+D\] If the concentration of A is doubled then the rate of reaction will

    A)
     remain unchanged

    B)
     become four times

    C)
     become two times

    D)
     become quarter

    View Answer play_arrow
  • question_answer109) For endothermic reaction, on increasing temperature the value of\[{{K}_{eq}}\]will

    A)
     increase

    B)
     decrease

    C)
     approximately remain unchanged

    D)
     None of the above

    View Answer play_arrow
  • question_answer110) The type of hybridization in\[C{{H}_{2}}=C=C{{H}_{2}}\]is

    A)
     only \[s{{p}^{2}}\]         

    B)
     only\[sp\]

    C)
    \[sp\]and\[s{{p}^{2}}\]       

    D)
     \[s{{p}^{2}}\]and\[s{{p}^{3}}\]

    View Answer play_arrow
  • question_answer111) Which of the following has the maximum electronegativity?

    A)
     F                

    B)
     \[Cl\]

    C)
     N                

    D)
     \[O\]

    View Answer play_arrow
  • question_answer112) Which of the following effect will observe, on passing light from colloidal solution?

    A)
     Electrophoresis   

    B)
     Tyndall effect

    C)
     Electro osmosis   

    D)
     Coagulation

    View Answer play_arrow
  • question_answer113) The order of the following reaction is three, \[6KI+2FeC{{l}_{3}}\xrightarrow[{}]{{}}2FeI+6KCl+2{{I}_{2}}\] What will be the molecularity of this reaction?

    A)
     3               

    B)
     8

    C)
    \[-1\]              

    D)
     2

    View Answer play_arrow
  • question_answer114) Which of the following indicator will be used in the following reaction? \[KMn{{O}_{4}}+F{{e}^{2+}}\xrightarrow[{}]{{}}\]Product

    A)
     Methyl red

    B)
     Phenolphthalein

    C)
     Phenol red

    D)
     Self indicator

    View Answer play_arrow
  • question_answer115) At constant temperature, the relationship between osmotic pressure and volume will be

    A)
     \[p\propto V\]

    B)
     \[p\propto \frac{1}{V}\]

    C)
     \[p\propto \sqrt{V}\]

    D)
     \[p\]

    View Answer play_arrow
  • question_answer116) The pH of a solution is 4.0. On adding phenolphthalein in this solution, the colour of solution will be

    A)
     pink             

    B)
     yellow

    C)
     red               

    D)
     colourless

    View Answer play_arrow
  • question_answer117) The nature of\[{{H}_{2}}{{O}_{2}}\] is

    A)
     weak basic       

    B)
     weak acidic

    C)
     strong acidic     

    D)
     strong basic

    View Answer play_arrow
  • question_answer118) The product will be obtained in the following reaction \[{{K}_{4}}[Fe{{(CN)}_{6}}]+{{O}_{3}}\xrightarrow[{}]{{{H}_{2}}O}\]Product

    A)
     \[F{{e}^{2+}}\]ion

    B)
     \[{{K}_{3}}[Fe{{(CN)}_{6}}]\]

    C)
     \[C{{N}^{-}}\]ion         

    D)
     None of these

    View Answer play_arrow
  • question_answer119) According to Pauli exclusion principle, any two electrons in an atom cannot have identical set of ....... quantum number.

    A)
     1                   

    B)
     2

    C)
     3                   

    D)
     4

    View Answer play_arrow
  • question_answer120) Magnetic moment of\[C{{u}^{+}}\]ion is

    A)
     1.73            

    B)
     0

    C)
     2.83            

    D)
     3.95

    View Answer play_arrow
  • question_answer121) Which of the following has\[1{{s}^{2}},2{{s}^{2}}2{{p}^{6}},\] \[3{{s}^{2}}3{{p}^{6}},3{{d}^{10}}4{{s}^{2}}\]electronic configuration?

    A)
     Zn              

    B)
     \[Cu\]

    C)
     \[Co\]               

    D)
     \[Ni\]

    View Answer play_arrow
  • question_answer122) Which of the following has zero bond order?

    A)
     \[He_{2}^{+}\]

    B)
     \[H{{e}_{2}}\]

    C)
     \[H_{2}^{+}\]

    D)
     \[H_{2}^{-}\]

    View Answer play_arrow
  • question_answer123) An element has the following characters (1) atomic number : 31 (2) with\[HN{{O}_{2}}:\]no reaction (3) reaction with\[{{O}_{2}}:\]no reaction (4) on heating form a layer of oxide (5) used in transistor then the element will be

    A)
     Ga              

    B)
    \[In\]

    C)
     Br               

    D)
    \[Mg\]

    View Answer play_arrow
  • question_answer124) Element\[Ca,Sr,Ba\]show colours in Bunsen's burner flame due to

    A)
     low ionization potential value

    B)
     emission and absorption of energy by electron

    C)
     high ionization potential value

    D)
     transfer of energy

    View Answer play_arrow
  • question_answer125) Decreasing order of radius of hydrated ions of \[Li,Na,K,Rb\]and Cs is

    A)
     \[Li>Na>K>Rb>Cs\]

    B)
     \[Cs>Rb>K>Na>Li\]

    C)
     \[Cs>Rb>K>Li>Na\]

    D)
     \[Cs>K>Rb>Na>Li\]

    View Answer play_arrow
  • question_answer126) If the frequency of sodium ion is\[5.09\times {{10}^{14}}{{s}^{-1}}\]then wavelength will be

    A)
     5.89nm        

    B)
     589 nm

    C)
     58.9 nm        

    D)
     5890 nm

    View Answer play_arrow
  • question_answer127) Electron deficient compound is

    A)
     \[PC{{l}_{3}}\]             

    B)
     \[BC{{l}_{3}}\]

    C)
     \[ICl\]             

    D)
    \[PC{{l}_{5}}\]

    View Answer play_arrow
  • question_answer128) In the following reaction, \[N{{H}_{3}}+B{{F}_{3}}\xrightarrow[{}]{{}}N{{H}_{3}}\to B{{F}_{3}}\] which behaves like Lewis base?

    A)
     \[N{{H}_{3}}\]             

    B)
     \[B{{F}_{3}}\]

    C)
     \[N{{H}_{3}}\to B{{F}_{3}}\]      

    D)
     None of these

    View Answer play_arrow
  • question_answer129) For\[{{H}_{2}}O\]and\[{{D}_{2}}O,\]the correct statement is

    A)
     Both have same boiling point

    B)
     Surface tension of\[{{D}_{2}}O\]is less than\[{{H}_{2}}O\]

    C)
     Viscosity of\[{{H}_{2}}O\]is greater than \[{{D}_{2}}O\]

    D)
     Solubility of\[{{D}_{2}}O\]is more in ionic compound

    View Answer play_arrow
  • question_answer130) When\[BaS\]reacts with\[{{D}_{2}}O\] then products obtained

    A)
     \[BaO+{{D}_{2}}S\]

    B)
     \[Ba{{(OD)}_{2}}+{{D}_{2}}S\]

    C)
     \[BaS{{O}_{4}}.{{D}_{2}}O+S\]

    D)
     \[Ba{{O}_{2}}+{{D}_{2}}O\]

    View Answer play_arrow
  • question_answer131) In the reaction,\[4LiH+AlC{{l}_{3}}\xrightarrow[{}]{Ether}\]Product. The product is

    A)
     \[LiCl\]              

    B)
     \[L{{i}_{2}}O\]

    C)
     \[LiAl{{H}_{4}}\]          

    D)
     \[Al{{H}_{3}}\]

    View Answer play_arrow
  • question_answer132) If mass of\[C{{O}_{2}}\]molecule is 44u and Avogadro's number is\[6.023\times {{10}^{23}}\]then mass of one molecule in kg is

    A)
     \[16\times {{10}^{-28}}kg\]     

    B)
     \[2.16\times {{10}^{-22}}kg\]

    C)
     \[3.46\times {{10}^{-28}}kg\]   

    D)
     \[7.3\times {{10}^{-26}}kg\]

    View Answer play_arrow
  • question_answer133) Molecular weight of a compound is 96.5 and has the following characters. (1) poisonous (2) isodiapher (3) finds in cis and trans form (4) hypnotic then the compound is

    A)
     \[{{C}_{2}}HC{{l}_{3}}\]         

    B)
     \[{{C}_{2}}{{H}_{2}}C{{l}_{2}}\]

    C)
     \[{{C}_{2}}{{H}_{3}}Cl\]         

    D)
     \[{{C}_{2}}C{{l}_{4}}\]

    View Answer play_arrow
  • question_answer134) Presence of three unpaired electrons in p-orbital of phosphorus can be explained by

    A)
     Aufbau rule      

    B)
     Hund's rule

    C)
     Faults rule       

    D)
    \[(n+7)\]rule

    View Answer play_arrow
  • question_answer135) Colloidal solution is not formed by

    A)
     precipitation

    B)
     peptisation

    C)
     double decomposition

    D)
     mechanical decomposition

    View Answer play_arrow
  • question_answer136) Formula of bauxite is

    A)
     \[A{{l}_{2}}{{O}_{3}}.2{{H}_{2}}O\]

    B)
     \[A{{l}_{2}}{{O}_{3}}.{{H}_{2}}O\]

    C)
     \[A{{l}_{2}}{{O}_{3}}\]

    D)
     \[A{{l}_{2}}{{O}_{3}}.3{{H}_{2}}O\]

    View Answer play_arrow
  • question_answer137) Decrease in oxidation number is called

    A)
     oxidation        

    B)
     reduction

    C)
     redox reaction    

    D)
     None of these

    View Answer play_arrow
  • question_answer138) For a buffer solution of\[C{{H}_{3}}COOH\]and \[C{{H}_{3}}COONa\]if\[[C{{H}_{3}}COOH],[C{{H}_{3}}COONa]=1\] and\[p{{K}_{a}}=4.74\]the pH value of the solution will be

    A)
     5.74              

    B)
     6.74

    C)
     3.74              

    D)
     4.74

    View Answer play_arrow
  • question_answer139) If\[pH=8\]solution is basic, then the nature of this solution will be corresponding to that solution whose\[pH=12\]

    A)
     less basic        

    B)
     more basic

    C)
     equal           

    D)
     None of these

    View Answer play_arrow
  • question_answer140) CNG is

    A)
    \[C{{H}_{4}}\] (84%) + propane + butane + higher alkane

    B)
    \[C{{H}_{4}}\](33%) + ethane (33%) + butane (33%)

    C)
     benzene (10%) + petrol (90%)

    D)
    \[C{{H}_{4}}\] (10%) + LPG (90%)

    View Answer play_arrow
  • question_answer141) Which of the following compound is strongly polar and has high dipole moment?

    A)
     \[CHC{{l}_{3}}\]

    B)
     \[C{{H}_{2}}C{{l}_{2}}\]

    C)
     \[CC{{l}_{4}}\]

    D)
     \[C{{O}_{2}}\]

    View Answer play_arrow
  • question_answer142) If the hybridization in\[{{C}_{2}}{{H}_{2}}\]is sp, then what will be the bond angle?

    A)
     \[180{}^\circ \]             

    B)
     \[120{}^\circ \]

    C)
     \[150{}^\circ \]             

    D)
     \[90{}^\circ \]

    View Answer play_arrow
  • question_answer143) Isoelectronic for\[{{K}^{+}}\] is

    A)
     \[Ar\]               

    B)
     \[Cl\]

    C)
     S                

    D)
     \[Ca\]

    View Answer play_arrow
  • question_answer144) When \[{{H}_{2}}\]reacts with\[C{{l}_{2}}\] in presence of sunlight to form \[HCl\]then order of reaction will be (unit of\[K=mol\,{{L}^{-1}}{{S}^{-1}}\])

    A)
     2                

    B)
     3

    C)
     0                

    D)
     4

    View Answer play_arrow
  • question_answer145) Correct order of ionization potential is

    A)
     \[B>Be\]           

    B)
     \[Be>B\]

    C)
     \[Be=B\]           

    D)
     None of these

    View Answer play_arrow
  • question_answer146) According to the reaction, \[Cu+N{{H}_{3}}\xrightarrow[{}]{Air}\] Product;, The product is

    A)
     \[C{{u}^{2+}}\]

    B)
     \[C{{u}^{2+}}.NO\]

    C)
     \[[Cu{{(N{{H}_{3}})}_{4}}]{{(OH)}_{2}}\]

    D)
     \[Cu{{(OH)}_{2}}\]

    View Answer play_arrow
  • question_answer147) Main ore of an iron is

    A)
     \[F{{e}_{3}}{{O}_{4}}\]           

    B)
     \[FeC{{O}_{3}}\]

    C)
     \[F{{e}_{2}}{{O}_{3}}\]           

    D)
     \[Fe{{S}_{2}}\]

    View Answer play_arrow
  • question_answer148) Diagonal relationship shows

    A)
    \[Ca\]and Ba       

    B)
     Be and Mg

    C)
     \[Na\]and K       

    D)
     Li and Be

    View Answer play_arrow
  • question_answer149) Number of crand n bonds in\[A-N=N-B\]

    A)
     \[3\sigma ,1\pi \]

    B)
     \[2\sigma ,0\pi \]

    C)
     \[2\sigma ,2\pi \]

    D)
     \[4\sigma ,0\pi \]

    View Answer play_arrow
  • question_answer150) Which of the following compound has zero dipole moment?

    A)
     \[C{{O}_{2}}\]            

    B)
     \[S{{O}_{2}}\]

    C)
     \[N{{H}_{3}}\]            

    D)
    \[{{H}_{2}}O\]

    View Answer play_arrow
  • question_answer151) If the value of velocity indices are 2 a, 3b and 1c then the value of Miller's indices will be

    A)
     3, 2, 6            

    B)
     2, 3, 1

    C)
     1, 2, 3            

    D)
     1, 1, 1

    View Answer play_arrow
  • question_answer152) Radius of\[_{6}{{C}^{12}}\]nucleus will be

    A)
     \[3.2\times {{10}^{-13}}cm\]   

    B)
     \[1.6\times {{10}^{-13}}cm\]

    C)
     \[4.8\times {{10}^{-13}}cm\]   

    D)
     \[6.4\times {{10}^{-3}}cm\]

    View Answer play_arrow
  • question_answer153) Electron affinity of nitrogen is

    A)
     0.2                

    B)
     0.002

    C)
     0.6                

    D)
     2

    View Answer play_arrow
  • question_answer154) If the dissociation constant of two acids are \[2.7\times {{10}^{-4}}\]and\[3\times {{10}^{-5}}\]then the ratio of their relative strength will be

    A)
     \[9:1\]              

    B)
     \[3:1\]

    C)
     \[3:2\]              

    D)
     \[9:2\]

    View Answer play_arrow
  • question_answer155) Colour of\[ZnS{{O}_{4}}\]is white. It is due to

    A)
     completely filled\[{{d}^{10}}\]orbitals

    B)
     magnetic moment\[\mu =0\]

    C)
     coupling of spin and transfer of electron

    D)
     pseudo inert configuration

    View Answer play_arrow
  • question_answer156) Solution of\[C{{H}_{3}}COON{{H}_{4}}\]is

    A)
     amphoteric       

    B)
     neutral

    C)
     basic             

    D)
     acidic

    View Answer play_arrow
  • question_answer157) At 373K value of\[{{K}_{sp}}\]of\[AgCl\]is\[1.44\times {{10}^{-4}},\] then the solubility will be

    A)
     \[1.2\times {{10}^{-2}}\]

    B)
     \[1.2\times {{10}^{-4}}\]

    C)
     \[72\times {{10}^{-2}}\]

    D)
     \[72\times {{10}^{-4}}\]

    View Answer play_arrow
  • question_answer158) Alcohol does not completely neutralize with which of the following?

    A)
     Vitamin         

    B)
     Amyi alcohol

    C)
     Castrol oil       

    D)
     Aniline

    View Answer play_arrow
  • question_answer159) On heating\[N{{H}_{3}}\]with ethyl acetate, then the product obtained will be will obtain

    A)
     ethyl amine      

    B)
     ethanamide

    C)
     ammonium acetate

    D)
     acetic acid

    View Answer play_arrow
  • question_answer160) For the polymerization of ethylene, the suitable condition is

    A)
     only high temperature

    B)
     only catalyst

    C)
     only high pressure

    D)
     high temperature and high pressure

    View Answer play_arrow
  • question_answer161) Keto-enol isomerism shows

    A)
     \[{{C}_{6}}{{H}_{5}}-\underset{\begin{smallmatrix}  || \\  O \end{smallmatrix}}{\mathop{C}}\,-C{{H}_{3}}\]

    B)
     \[{{C}_{6}}{{H}_{5}}-\underset{\begin{smallmatrix}  || \\  O \end{smallmatrix}}{\mathop{C}}\,-{{C}_{6}}{{H}_{5}}\]

    C)
     \[{{C}_{6}}{{H}_{5}}-\underset{\begin{smallmatrix}  || \\  O \end{smallmatrix}}{\mathop{C}}\,-C{{(C{{H}_{3}})}_{3}}\]

    D)
     \[{{C}_{6}}{{H}_{5}}-\underset{\begin{smallmatrix}  || \\  O \end{smallmatrix}}{\mathop{C}}\,-H\]

    View Answer play_arrow
  • question_answer162) In the presence of KOH, phenol reacts with\[{{K}_{2}}{{S}_{2}}{{O}_{8}}\]to form the product

    A)
     benzene

    B)
     1, 4-dihydroxy benzene

    C)
     benzoic acid

    D)
     4-hydroxy benzoic acid

    View Answer play_arrow
  • question_answer163) Polyacetylene is used

    A)
     in the formation of medicine

    B)
     in the formation of solid fuel

    C)
     in the formation of insecticide

    D)
     it is not possible

    View Answer play_arrow
  • question_answer164) On the reaction of acetone and phenyl hydrazine the product will form

    A)
     \[C{{H}_{3}}C{{H}_{2}}C{{H}_{2}}-N{{H}_{2}}\]

    B)
     \[C{{H}_{3}}-\overset{\begin{smallmatrix}  C{{H}_{3}} \\  | \end{smallmatrix}}{\mathop{C}}\,=N-\overset{\begin{smallmatrix}  H \\  | \end{smallmatrix}}{\mathop{N}}\,-{{C}_{6}}{{H}_{5}}\]

    C)
     \[C{{H}_{3}}-\overset{\begin{smallmatrix}  C{{H}_{3}} \\  | \end{smallmatrix}}{\mathop{C}}\,=N-N{{H}_{2}}\]

    D)
     None of the above

    View Answer play_arrow
  • question_answer165) On the reaction of butyric acid with\[{{H}_{2}}{{O}_{2}},\]the product will form

    A)
     \[C{{H}_{3}}C{{H}_{2}}CHO\]

    B)
     \[\underset{\begin{smallmatrix}  | \\  OH \end{smallmatrix}}{\mathop{C}}\,-C{{H}_{2}}-C{{H}_{2}}-\underset{\begin{smallmatrix}  || \\  O \end{smallmatrix}}{\mathop{C}}\,-OH\]

    C)
     \[C{{H}_{3}}-C{{H}_{2}}-\underset{\begin{smallmatrix}  | \\  OH \end{smallmatrix}}{\mathop{CH}}\,-\underset{\begin{smallmatrix}  || \\  O \end{smallmatrix}}{\mathop{C}}\,-OH\]

    D)
     \[C{{H}_{3}}-\underset{\begin{smallmatrix}  | \\  OH \end{smallmatrix}}{\mathop{CH}}\,-C{{H}_{2}}-\underset{\begin{smallmatrix}  || \\  O \end{smallmatrix}}{\mathop{C}}\,-OH\]

    View Answer play_arrow
  • question_answer166) Acid in which\[COOH\]group is not present is

    A)
     aspirin           

    B)
     anthranilic acid

    C)
     picric acid       

    D)
     amino acid

    View Answer play_arrow
  • question_answer167) Sodium lauryl sulphate is

    A)
     cationic detergent

    B)
     anionic detergent

    C)
     foam active       

    D)
     neutral detergent

    View Answer play_arrow
  • question_answer168) \[2{{C}_{6}}{{H}_{5}}N{{H}_{2}}+C{{S}_{2}}+2KOH\xrightarrow[{}]{{}}\] Main product, the main product will be

    A)
     \[{{C}_{6}}{{H}_{5}}-N=C=S\]

    B)
     

    C)
     \[{{C}_{6}}{{H}_{5}}-\underset{\begin{smallmatrix}  || \\  O \end{smallmatrix}}{\mathop{C}}\,-N{{H}_{2}}\]

    D)
     None of these

    View Answer play_arrow
  • question_answer169) In the reaction, \[6C{{H}_{2}}=C{{H}_{2}}+{{B}_{2}}{{H}_{6}}\xrightarrow[{}]{{}}2\left( H-\underset{\begin{smallmatrix}  | \\  H \end{smallmatrix}}{\overset{\begin{smallmatrix}  H \\  | \end{smallmatrix}}{\mathop{C}}}\,-\underset{\begin{smallmatrix}  | \\  H \end{smallmatrix}}{\overset{\begin{smallmatrix}  H \\  | \end{smallmatrix}}{\mathop{C}}}\, \right)-B\] the carb-boron product known as alkyl borane, on oxidation gives alcohols. This was made by which scientist?

    A)
     Brown and Benzamine

    B)
     Jvifail

    C)
     Methason

    D)
     Suberao

    View Answer play_arrow
  • question_answer170) In the presence of sunlight, when hot toluene reacts with excess of\[C{{l}_{2}},\]the product will be

    A)
     chloro benzene   

    B)
     benzal chloride

    C)
     benzyl chloride  

    D)
     benzo trichloride

    View Answer play_arrow
  • question_answer171) Monomer of natural rubber is

    A)
     \[C{{H}_{2}}=C=C{{H}_{2}}\]

    B)
     \[C{{H}_{2}}=CH-Cl\]

    C)
     \[C{{H}_{2}}=\underset{\begin{smallmatrix}  | \\  C{{H}_{3}} \end{smallmatrix}}{\mathop{C}}\,-CH=C{{H}_{2}}\]

    D)
     None of the above

    View Answer play_arrow
  • question_answer172) Monomers of nylon-6 6 are

    A)
     isoprene + chloroprene

    B)
     adipic acid + ethylene glycol

    C)
     adipic acid + hexamethylene diamine

    D)
     None of the above

    View Answer play_arrow
  • question_answer173) Nylon-6 is formed from caprolactum. Formula of caprolactum is

    A)
     

    B)
     

    C)
     

    D)
    \[COOH{{(C{{H}_{2}})}_{6}}COOH\]

    View Answer play_arrow
  • question_answer174) In the esterification of alcohol, the order of reactivity is

    A)
     \[2{}^\circ >3{}^\circ >1{}^\circ \]

    B)
     \[3{}^\circ >2{}^\circ >1{}^\circ \]

    C)
     \[1{}^\circ >2{}^\circ >3{}^\circ \]

    D)
     \[1{}^\circ >2{}^\circ >3{}^\circ \]

    View Answer play_arrow
  • question_answer175) 1, 3-pentadiene   is   more   stable   than 1, 4-pentadiene because

    A)
     it is a conjugated diene

    B)
     its dipole moment is more

    C)
     Both are functional and position isomers

    D)
     None of the above

    View Answer play_arrow
  • question_answer176) On complete oxidation ether gives

    A)
     \[{{C}_{2}}H.pH\]        

    B)
    \[{{C}_{2}}{{H}_{2}}\]

    C)
     \[C{{O}_{2}}+{{H}_{2}}O\]      

    D)
     \[C{{H}_{3}}COOH\]

    View Answer play_arrow
  • question_answer177) In the electrolysis of sodium acetate,\[{{H}_{2}}\]gas releases at cathode and\[{{C}_{2}}{{H}_{6}}\]gas at anode. This reaction is known as

    A)
     Frenkland       

    B)
     Kolbe

    C)
     Clemmensen     

    D)
     Wolff-Kishner

    View Answer play_arrow
  • question_answer178) Reactivity order of HX with alcohol is

    A)
     \[HCl>HBr>HI>HF\]

    B)
     \[HF>HCl>HBr>HI\]

    C)
     \[HI>HCl>HBr>HF\]

    D)
     \[HI>HBr>HCl>HF\]

    View Answer play_arrow
  • question_answer179) Anhydrous formic acid cannot be obtained from fractional distillation of aqueous HCOOH because

    A)
     it forms complex with water

    B)
     its boiling point is 373.4 K

    C)
     it is soluble in water

    D)
     None of the above

    View Answer play_arrow
  • question_answer180) TEL added in petrol and other fuels because

    A)
     it reduces heat

    B)
     it reduces coolness

    C)
     it reduces pressure

    D)
     it reduces knocking property

    View Answer play_arrow
  • question_answer181) Which of the following shows optical isomerism?

    A)
     \[C{{H}_{3}}-\underset{\begin{smallmatrix}  | \\  Cl \end{smallmatrix}}{\overset{\begin{smallmatrix}  Br \\  | \end{smallmatrix}}{\mathop{C}}}\,-C{{H}_{3}}\]

    B)
    \[C{{H}_{3}}-\underset{\begin{smallmatrix}  | \\  Cl \end{smallmatrix}}{\overset{\begin{smallmatrix}  Br \\  | \end{smallmatrix}}{\mathop{C}}}\,-\underset{\begin{smallmatrix}  || \\  O \end{smallmatrix}}{\mathop{C}}\,-OH\]

    C)
     \[H-\underset{\begin{smallmatrix}  | \\  Cl \end{smallmatrix}}{\overset{\begin{smallmatrix}  Br \\  | \end{smallmatrix}}{\mathop{C}}}\,-H\]

    D)
    \[C{{H}_{3}}-C{{H}_{2}}-\underset{\begin{smallmatrix}  || \\  O \end{smallmatrix}}{\mathop{C}}\,-OH\]

    View Answer play_arrow
  • question_answer182) Which of the following reagent used in the formation of ethane amine from propanamide?

    A)
     \[B{{r}_{2}}+KOH\]

    B)
     \[Na+{{C}_{2}}{{H}_{5}}OH\]

    C)
     \[LiAl{{H}_{4}}\]

    D)
     \[AlC{{l}_{3}}\]

    View Answer play_arrow
  • question_answer183) In the reaction. \[HCOOH+C{{H}_{3}}COOH\xrightarrow[\Delta ]{MnO}X;\] Product X is

    A)
     \[HCHO\]

    B)
     \[C{{H}_{3}}-\underset{\begin{smallmatrix}  || \\  O \end{smallmatrix}}{\mathop{C}}\,-H\]

    C)
     \[C{{H}_{3}}-\underset{\begin{smallmatrix}  || \\  O \end{smallmatrix}}{\mathop{C}}\,-O-{{C}_{2}}{{H}_{5}}\]

    D)
     \[H-\underset{\begin{smallmatrix}  || \\  O \end{smallmatrix}}{\mathop{C}}\,-O-{{C}_{2}}{{H}_{5}}\]

    View Answer play_arrow
  • question_answer184) \[{{C}_{6}}{{H}_{5}}OH\]and\[{{C}_{6}}{{H}_{5}}COOH\]can be distinguished by

    A)
     \[FeC{{l}_{3}}\]           

    B)
     Na metal

    C)
     \[NaOH\]           

    D)
     litmus

    View Answer play_arrow
  • question_answer185) For increase in strong basic nature of aniline, which group should attach on para position?

    A)
     \[-OC{{H}_{3}}\]          

    B)
     \[-C{{H}_{3}}\]

    C)
     \[-N{{O}_{2}}\]           

    D)
     \[-Cl\]

    View Answer play_arrow
  • question_answer186) Which of the following alkene gives acetone on ozonolysis?

    A)
     2-methyl propene

    B)
     2-methyl-l-butene

    C)
     2-butene         

    D)
     Ethene

    View Answer play_arrow
  • question_answer187) Which of the following cannot be obtained by\[CHC{{l}_{3}}\]?

    A)
     Freon            

    B)
     Chloropicrin

    C)
     Chloretone       

    D)
     Salicyldehyde

    View Answer play_arrow
  • question_answer188) Which of the following doesn't show Friedel-Craft reaction?

    A)
     Nitrobenzene    

    B)
     Toluene

    C)
     Benzene         

    D)
     All of these

    View Answer play_arrow
  • question_answer189) Increasing reactivity order of nucleophilic addition reaction of \[H-\underset{\begin{smallmatrix}  || \\  O \end{smallmatrix}}{\mathop{C}}\,-H;C{{H}_{3}}-\underset{\begin{smallmatrix}  || \\  O \end{smallmatrix}}{\mathop{C}}\,-H\] and\[C{{H}_{3}}-\underset{\begin{smallmatrix}  || \\  O \end{smallmatrix}}{\mathop{C}}\,-C{{H}_{3}}\]will be

    A)
     \[HCHO>C{{H}_{3}}CHO>C{{H}_{3}}COC{{H}_{3}}\]

    B)
     \[C{{H}_{3}}CHO>HCHO>C{{H}_{3}}COC{{H}_{3}}\]

    C)
     \[C{{H}_{3}}COC{{H}_{3}}>C{{H}_{3}}CHO>HCHO\]

    D)
     \[C{{H}_{3}}CHO>C{{H}_{3}}COC{{H}_{3}}>HCHO\]

    View Answer play_arrow
  • question_answer190) Purity of organic compound is determined by

    A)
     molecular weight

    B)
     boiling point

    C)
     density

    D)
     solubility in water

    View Answer play_arrow
  • question_answer191) Which of the following reaction is not stereospecific?

    A)
     Electrophilic substitution

    B)
     \[{{S}_{N}}1\]

    C)
     \[{{S}_{N}}2\]

    D)
     Addition of\[B{{r}_{2}}\]on ethylene

    View Answer play_arrow
  • question_answer192) The product will be obtained from the reaction of\[{{C}_{2}}{{H}_{2}}\]with\[AsC{{l}_{3}}\]in the presence of\[(AlC{{l}_{3}}+HCl)\]

    A)
     Lewisite

    B)
     Electrophile

    C)
     Insecticide

    D)
     Nucleophite

    View Answer play_arrow
  • question_answer193) \[R-C\equiv N\xrightarrow[{}]{Sn+HCl}X\xrightarrow[{}]{{{H}_{2}}O}Y\]reaction is known as

    A)
     Rosenmund's reaction

    B)
     Stephen's reaction

    C)
     Clemmensen reduction

    D)
     Cannizaro reaction

    View Answer play_arrow
  • question_answer194) \[C{{H}_{3}}-Mg-X+H-\underset{\begin{smallmatrix}  || \\  O \end{smallmatrix}}{\mathop{C}}\,-H\xrightarrow[{}]{{}}X\xrightarrow[{}]{{{H}_{2}}O}Y\] Product is

    A)
     \[{{C}_{2}}{{H}_{5}}OH\]

    B)
     \[C{{H}_{3}}CHO\]

    C)
     \[C{{H}_{3}}COOH\]

    D)
     \[C{{H}_{3}}-\underset{\begin{smallmatrix}  || \\  O \end{smallmatrix}}{\mathop{C}}\,-O-{{C}_{2}}{{H}_{5}}\]

    View Answer play_arrow
  • question_answer195) \[2C{{H}_{3}}-\underset{\begin{smallmatrix}  || \\  O \end{smallmatrix}}{\mathop{C}}\,-H\xrightarrow[{}]{Dil.base}X\xrightarrow[{}]{\Delta }Y\] Product ?Y? is

    A)
     \[C{{H}_{3}}-CH=CH-\underset{\begin{smallmatrix}  || \\  O \end{smallmatrix}}{\mathop{C}}\,-H\]

    B)
     \[C{{H}_{3}}-\underset{\begin{smallmatrix}  | \\  OH \end{smallmatrix}}{\mathop{CH}}\,-C{{H}_{2}}-\underset{\begin{smallmatrix}  || \\  O \end{smallmatrix}}{\mathop{C}}\,-H\]

    C)
     \[C{{H}_{3}}-CH=CH-COOH\]

    D)
     \[C{{H}_{3}}-\underset{\begin{smallmatrix}  | \\  OH \end{smallmatrix}}{\mathop{CH}}\,-C{{H}_{2}}-COOH\]

    View Answer play_arrow
  • question_answer196) \[2Cr{{(OH)}_{3}}+4O{{H}^{-}}+KI{{O}_{3}}\to 2CrO_{4}^{2-}\]\[+5{{H}_{2}}O+KI\] Equivalent weight of\[KI{{O}_{3}}\]in the above reaction is

    A)
     molecular weight

    B)
     \[\frac{molecular\text{ }weight}{3}\]

    C)
     \[\frac{molecular\text{ }weight}{6}\]  

    D)
     \[\frac{molecular\text{ }weight}{2}\]

    View Answer play_arrow
  • question_answer197) Which metal is present in chlorophyll?

    A)
     Cr               

    B)
     Co

    C)
     Mg              

    D)
     Fe

    View Answer play_arrow
  • question_answer198) Which gas is used for the cutting and welding of metal?

    A)
     Acetylene        

    B)
     Methane

    C)
     Carbon dioxide   

    D)
     Neon

    View Answer play_arrow
  • question_answer199) Which of the following will give silver mirror test?

    A)
     Glucose                                   

    B)
     Fructose

    C)
     Starch           

    D)
     Sucrose

    View Answer play_arrow
  • question_answer200) Conjugated acid of\[NH_{2}^{-}\]is

    A)
     \[NH_{4}^{+}\]            

    B)
     \[N{{H}_{4}}OH\]

    C)
     \[N{{H}_{3}}\]             

    D)
     \[{{N}_{2}}{{O}_{4}}\]

    View Answer play_arrow
  • question_answer201) \[\underset{x\to 0}{\mathop{\lim }}\,\frac{{{e}^{\tan x}}-{{e}^{x}}}{\tan x-x}\]is equal to

    A)
     0

    B)
     1

    C)
     \[e\]

    D)
     \[1/e\]

    View Answer play_arrow
  • question_answer202) \[\underset{x\to 0}{\mathop{\lim }}\,\frac{\sqrt{\frac{1-\cos 2x}{2}}}{x}\]is equal to

    A)
     1             

    B)
     \[-1\]

    C)
     0                

    D)
     None of these

    View Answer play_arrow
  • question_answer203) \[\underset{x\to 0}{\mathop{\lim }}\,\frac{x}{{{\tan }^{-1}}2x}\]is equal to

    A)
     2

    B)
     \[\frac{1}{2}\]

    C)
     0                

    D)
     None of these

    View Answer play_arrow
  • question_answer204) If\[y=a{{x}^{n+1}}+b{{x}^{-n}},\]then\[\frac{{{x}^{2}}{{d}^{2}}y}{d{{x}^{2}}}\]is equal to

    A)
     \[ny\]

    B)
     \[{{n}^{2}}y\]

    C)
     \[n(n-1)y\]

    D)
     \[n(n+1)y\]

    View Answer play_arrow
  • question_answer205) \[\frac{d}{dx}\left[ \log \left\{ {{e}^{x}}{{\left( \frac{x-2}{x+2} \right)}^{3/4}} \right\} \right]\]is equal to

    A)
     1

    B)
     \[\frac{{{x}^{2}}+1}{{{x}^{2}}-4}\]

    C)
     \[\frac{{{x}^{2}}-1}{{{x}^{2}}-4}\]

    D)
     \[\frac{{{x}^{2}}-1}{{{x}^{2}}-4}{{e}^{x}}\]

    View Answer play_arrow
  • question_answer206) \[\int{\frac{1+{{\tan }^{2}}x}{1-{{\tan }^{2}}x}}dx\]is equal to

    A)
     \[\log \left( \frac{1-\tan x}{1+\tan x} \right)+c\]

    B)
     \[\log \left( \frac{1+\tan x}{1-\tan x} \right)+c\]

    C)
     \[\frac{1}{2}\log \left( \frac{1-\tan x}{1+\tan x} \right)+c\]

    D)
     \[\frac{1}{2}\log \left( \frac{1+\tan x}{1-\tan x} \right)+c\]

    View Answer play_arrow
  • question_answer207) If\[{{a}^{2}}{{x}^{4}}+{{b}^{2}}{{y}^{4}}={{c}^{6}},\]then maximum value of\[xy\] is

    A)
     \[\frac{{{c}^{3}}}{2ab}\]

    B)
     \[\frac{{{c}^{3}}}{\sqrt{2ab}}\]

    C)
     \[\frac{{{c}^{3}}}{ab}\]

    D)
     \[\frac{{{c}^{3}}}{\sqrt{ab}}\]

    View Answer play_arrow
  • question_answer208) Maximum value of\[px+qy,\]when\[xy={{r}^{2}},\]is

    A)
     \[2r\sqrt{pq}\]

    B)
     \[2pq\sqrt{r}\]

    C)
     \[-2r\sqrt{pq}\]

    D)
     None of these

    View Answer play_arrow
  • question_answer209) If\[f(x)={{x}^{2}}+1,\] then the value of\[{{f}^{-1}}(17)\]and\[{{f}^{-1}}(-3)\]are respectively

    A)
     4, 1              

    B)
     4, 0

    C)
     3, 2              

    D)
     None of these

    View Answer play_arrow
  • question_answer210) If\[a=\cos \alpha +i\sin \alpha ,\]\[b=\cos \beta +i\sin \beta ,\]\[c=\cos \gamma +i\sin \gamma \]and\[\frac{b}{c}+\frac{c}{a}+\frac{a}{b}=1\]then \[\cos (\beta -\gamma )+\cos (\gamma -\alpha )+\cos (\alpha -\beta )\]is equal to

    A)
     \[\frac{3}{2}\]

    B)
     \[-\frac{3}{2}\]

    C)
     \[0\]

    D)
     1

    View Answer play_arrow
  • question_answer211) \[{{\left[ \frac{1+\cos \frac{\pi }{8}+i\sin \frac{\pi }{8}}{1+\cos \frac{\pi }{8}-i\sin \frac{\pi }{8}} \right]}^{8}}\]is equal to

    A)
     0              

    B)
     1

    C)
     \[-1\]             

    D)
     2

    View Answer play_arrow
  • question_answer212) \[{{i}^{n}}+{{i}^{n+1}}+{{i}^{n+2}}+{{i}^{n+3}},(n\in N)\]is equal to

    A)
     4               

    B)
     1

    C)
     0               

    D)
     2

    View Answer play_arrow
  • question_answer213) If\[2(y-a)\]is the harmonic mean of\[y-x\]and \[y-z,\]then\[x-a,\text{ }y-a,\text{ }z-a\]are in

    A)
     AP              

    B)
     GP

    C)
     HP              

    D)
     None of these

    View Answer play_arrow
  • question_answer214) If\[{{a}^{x}}={{b}^{y}}={{c}^{z}}={{d}^{u}}\]and a, b, c, d are in GP, then\[x,\text{ }y,\text{ }z\]are in

    A)
     AP             

    B)
     GP

    C)
     HP             

    D)
     None of these

    View Answer play_arrow
  • question_answer215) \[lo{{g}_{3}}2,\text{ }lo{{g}_{6}}2,\text{ }lo{{g}_{12}}2\]are in

    A)
     AP              

    B)
     GP

    C)
     HP              

    D)
     None of these

    View Answer play_arrow
  • question_answer216) Angle between the planes\[2x-y+z=6\]and \[x+y+2z=3\]is

    A)
     \[\frac{\pi }{3}\]

    B)
     \[\frac{\pi }{6}\]

    C)
     \[\frac{\pi }{2}\]             

    D)
     0

    View Answer play_arrow
  • question_answer217)  \[\int_{0}^{\pi /2}{\frac{\sqrt[3]{{{\sin }^{2}}x}dx}{\sqrt[3]{{{\sin }^{2}}x}\sqrt[3]{{{\cos }^{2}}x}}}\]is equal to

    A)
     \[\frac{\pi }{4}\]

    B)
     \[\frac{\pi }{2}\]

    C)
     \[\pi \]

    D)
     \[2\pi \]

    View Answer play_arrow
  • question_answer218) \[\int_{-1}^{1}{|1-x|}\,dx\]is equal to

    A)
     0

    B)
     \[-2\]

    C)
     4

    D)
     2

    View Answer play_arrow
  • question_answer219) \[\int_{0}^{1}{\frac{{{\tan }^{-1}}xdx}{1+{{x}^{2}}}}\]is equal to

    A)
     \[\frac{\pi }{4}\]

    B)
     \[\frac{\pi }{8}\]

    C)
     \[\frac{{{\pi }^{2}}}{16}\]

    D)
     \[\frac{{{\pi }^{2}}}{32}\]

    View Answer play_arrow
  • question_answer220) Area of the region bounded by the curve \[y=x,\text{ }x-\]axis and abscissas\[x=-1\]and\[x=2\]is

    A)
     0 sq unit         

    B)
     3/2 sq unit

    C)
     5/2 sq units      

    D)
     - 5/2 sq unit

    View Answer play_arrow
  • question_answer221) The value of \[\underset{x\to 1}{\mathop{\lim }}\,(1-x)\tan \frac{\pi x}{2}\]is equal to

    A)
     \[1\]

    B)
     \[\infty \]

    C)
     \[\frac{2}{\pi }\]

    D)
     0

    View Answer play_arrow
  • question_answer222) If\[z=i\log (2-\sqrt{3}),\]then\[cos\text{ }z\]is equal to

    A)
     0                 

    B)
     2

    C)
     4                 

    D)
     \[-2\]

    View Answer play_arrow
  • question_answer223) Tangent and normal at any point P of the parabola meet the axes at T and G respectively, then

    A)
     \[ST=SG.SP\]     

    B)
     \[ST\ne SG=SP\]

    C)
     \[ST=SG\ne SP\]    

    D)
     \[ST=SG=SP\]

    View Answer play_arrow
  • question_answer224) Domain of the function\[f(x)=\log (\sqrt{x-4}+\]\[\sqrt{6-x})\]is

    A)
     \[[4,\infty ]\]

    B)
     \[(-\infty ,6)\]

    C)
     [4, 6]             

    D)
     None of these

    View Answer play_arrow
  • question_answer225) Inverse of the function\[\frac{{{10}^{x}}-{{10}^{-x}}}{{{10}^{x}}+{{10}^{-x}}}\]is

    A)
     \[{{\log }_{10}}(2-x)\]

    B)
     \[\frac{1}{2}{{\log }_{10}}\left( \frac{1+x}{1-x} \right)\]

    C)
     \[\frac{1}{2}{{\log }_{10}}(2x-1)\]

    D)
     \[\frac{1}{4}{{\log }_{10}}\left( \frac{2x}{2-x} \right)\]

    View Answer play_arrow
  • question_answer226) Domain of the function \[f(x)={{\log }_{3+x}}({{x}^{2}}-1)\]is

    A)
     \[(-3,-1)\cup (1,\infty )\]

    B)
     \[(-3,-1)\cup [1,\infty )\]

    C)
     \[(-3,-2)\cup (-2,-1)\cup (1,\infty )\]

    D)
     \[[-3,-2)\cup (-2,-1)\cup [1,\infty )\]

    View Answer play_arrow
  • question_answer227) If\[n=3k\]and\[1,\omega ,{{\omega }^{2}}\]are the cube roots of unity, then   the   value   of  the   determinant \[\left| \begin{matrix}    1 & {{\omega }^{n}} & {{\omega }^{2n}}  \\    {{\omega }^{2n}} & 1 & {{\omega }^{n}}  \\    {{\omega }^{n}} & {{\omega }^{2n}} & 1  \\ \end{matrix} \right|\]is

    A)
     0                     

    B)
     \[\omega \]

    C)
     \[{{\omega }^{2}}\]                   

    D)
     1

    View Answer play_arrow
  • question_answer228) One root of the equation \[\left| \begin{matrix}    3-x & -6 & 3  \\    -6 & 3-x & 3  \\    3 & 3 & -6-x  \\ \end{matrix} \right|=0\]is

    A)
     6               

    B)
     3

    C)
     0               

    D)
     None of these

    View Answer play_arrow
  • question_answer229) Three coins are tossed together, then the probability of getting atleast one tail is

    A)
     \[\frac{1}{8}\]

    B)
     \[\frac{7}{8}\]

    C)
     \[\frac{1}{2}\]

    D)
     \[\frac{3}{4}\]

    View Answer play_arrow
  • question_answer230) A card is drawn randomly from a pack of 52 cards, then the probability that the card is king or spade is

    A)
     \[\frac{4}{13}\]

    B)
     \[\frac{1}{2}\]

    C)
     \[\frac{1}{4}\]

    D)
     \[\frac{3}{4}\]

    View Answer play_arrow
  • question_answer231) Angle between those two lines, in which direction   comes   are   represented   by \[l+m+n=0\]and\[{{l}^{2}}+{{m}^{2}}-{{n}^{2}}=0\]is

    A)
     \[\frac{2\pi }{3}\]

    B)
     \[\frac{\pi }{6}\]

    C)
     \[\frac{5\pi }{6}\]

    D)
     \[\frac{\pi }{3}\]

    View Answer play_arrow
  • question_answer232) If \[P(A)=0.25,P(B)=0.50\]and \[P(B/A)=0.6,\]then the value of P(A/B) is

    A)
     0.3                

    B)
     0.35

    C)
     0.15              

    D)
     0.5

    View Answer play_arrow
  • question_answer233) If \[f(x)=|x{{|}^{3}},\]then\[f'(0)\]is equal to

    A)
     0                   

    B)
     1

    C)
     \[f'(1)\]

    D)
     \[f'\,'(1)\]

    View Answer play_arrow
  • question_answer234) \[{{\cosh }^{-1}}x\]is equal to

    A)
     \[\log (x+\sqrt{{{x}^{2}}+1})\]

    B)
     \[\log (x+\sqrt{{{x}^{2}}-1})\]

    C)
     \[\log (x+\sqrt{1-{{x}^{2}}})\]

    D)
     \[\log (x-\sqrt{{{x}^{2}}-1})\]

    View Answer play_arrow
  • question_answer235) There are 16 points in a plane, out of which 6 points are collinear. Then, how many straight lines can be drawn using these lines.

    A)
     106             

    B)
     105

    C)
     60               

    D)
     55

    View Answer play_arrow
  • question_answer236) \[{{(1+i)}^{8}}+{{(1-i)}^{8}}\]is equal to

    A)
     16            

    B)
     \[-16\]

    C)
     32            

    D)
     \[-32\]

    View Answer play_arrow
  • question_answer237) \[\int{\frac{{{e}^{m{{\tan }^{-1}}x}}}{1+{{x}^{2}}}}dx\]is equal to

    A)
     \[{{e}^{m+{{\tan }^{-1}}x}}\]

    B)
     \[{{e}^{{{\tan }^{-1}}x}}\]

    C)
     \[\frac{1}{m}{{\tan }^{-1}}x\]

    D)
     None of these

    View Answer play_arrow
  • question_answer238) The adjoint matrix of the matrix\[\left[ \begin{matrix}    1 & 8  \\    2 & -5  \\ \end{matrix} \right]\]is given by

    A)
     \[\left[ \begin{matrix}    -5 & -8  \\    -2 & 1  \\ \end{matrix} \right]\]

    B)
     \[\left[ \begin{matrix}    -5 & -2  \\    -8 & 1  \\ \end{matrix} \right]\]

    C)
     \[\left[ \begin{matrix}    5 & -8  \\    -2 & 1  \\ \end{matrix} \right]\]

    D)
     \[\left[ \begin{matrix}    5 & -8  \\    -2 & -1  \\ \end{matrix} \right]\]

    View Answer play_arrow
  • question_answer239) If p and q are the roots of the equation \[{{x}^{2}}+px+q=0,\]then

    A)
     \[p=1\]             

    B)
     \[p=-2\]

    C)
     \[p=0\]and 1       

    D)
    \[p=-2\]and 0

    View Answer play_arrow
  • question_answer240) Coefficient of\[{{x}^{4}}\]in the expansion of \[{{(1+x+{{x}^{2}}+{{x}^{3}})}^{n}}\]is

    A)
     \[^{n}{{C}_{4}}\]

    B)
     \[^{n}{{C}_{4}}{{+}^{n}}{{C}_{2}}\]

    C)
     \[^{n}{{C}_{4}}{{+}^{n}}{{C}_{2}}{{+}^{n}}{{C}_{4}}^{n}{{C}_{2}}\]

    D)
     \[^{n}{{C}_{4}}{{+}^{n}}{{C}_{2}}{{+}^{n}}{{C}_{1}}^{n}{{C}_{2}}\]

    View Answer play_arrow
  • question_answer241) \[\int_{0}^{\pi /2}{\log \tan xdx}\]is equal to

    A)
     \[\frac{\pi }{4}\]

    B)
     \[\frac{\pi }{2}\]

    C)
     0                

    D)
     None of these

    View Answer play_arrow
  • question_answer242) Focus and directrix of the parabola\[{{x}^{2}}=-8ay\]are

    A)
    \[(-2a,0)\]and\[x=4\] 

    B)
    \[(0,2a)\]and\[y=-2a\]

    C)
    \[(0,-2a)\]and\[y=2a\]

    D)
    \[[2a,0)\]and\[x=-2a\]

    View Answer play_arrow
  • question_answer243) Coefficient of\[{{x}^{4}}\]in the expansion of \[{{\left( \frac{x}{2}-\frac{3}{{{x}^{2}}} \right)}^{10}}\]is

    A)
     \[\frac{504}{259}\]

    B)
     \[\frac{450}{263}\]

    C)
     \[\frac{405}{256}\]

    D)
     None of these

    View Answer play_arrow
  • question_answer244) Coordinates of that point on\[x-\]axis which is at a perpendicular distance 'a' from the line \[\frac{x}{a}+\frac{y}{b}=1,\]are

    A)
     \[(\pm a,0)\]

    B)
    \[\left( a\pm \frac{a}{b}\sqrt{{{a}^{2}}+{{b}^{2}}},0 \right)\]

    C)
     \[\left( a+\frac{a}{b}\sqrt{{{a}^{2}}+{{b}^{2}}},0 \right)\]

    D)
     \[\left( a-\frac{a}{b}\sqrt{{{a}^{2}}+{{b}^{2}}},0 \right)\]

    View Answer play_arrow
  • question_answer245) Equation of tangent to the circle\[{{x}^{2}}+{{y}^{2}}={{a}^{2}}\] which is parallel to the line\[y=mx+c,\]is

    A)
     \[my=x\pm a\sqrt{1+{{m}^{2}}}\]

    B)
     \[y=mx\pm \sqrt{1+{{m}^{2}}}\]

    C)
     \[y=mx\pm a\sqrt{1+{{m}^{2}}}\]

    D)
     None of these

    View Answer play_arrow
  • question_answer246) Which one of the following is correct?

    A)
     \[cos\text{ }ix=i\text{ }cosh\text{ }x\]  

    B)
     \[\tan ix=tanh\text{ }x\]

    C)
     \[\sin \text{ }ix=i\text{ }\sin \text{ }x\]    

    D)
     \[cos\text{ }ix=cosh\text{ }x\]

    View Answer play_arrow
  • question_answer247) Equation of line perpendicular to\[x=c\]is

    A)
     \[y=0\]             

    B)
     \[x=0\]

    C)
     \[x=y\]            

    D)
     \[x=d\]

    View Answer play_arrow
  • question_answer248) \[{{(\sin \theta +i\cos \theta )}^{n}}\] is equal to

    A)
     \[{{i}^{n}}(i\sin \theta +\cos \theta )\]

    B)
     \[(\sin n\theta +i\cos n\theta )\]

    C)
     \[\cos n\left( \frac{\pi }{2}-\theta  \right)+i\sin n\left( \frac{\pi }{2}-\theta  \right)\]

    D)
     \[\cos \theta +i\sin n\theta \]

    View Answer play_arrow
  • question_answer249) If A is a square matrix, then\[A+{{A}^{T}}\]is

    A)
     symmetric matrix

    B)
     anti-symmetric matrix

    C)
     diagonal matrix

    D)
     triangular matrix

    View Answer play_arrow
  • question_answer250) If a, b are two constant positive integers such that\[f(a+x)=b+[{{b}^{3}}+1-3{{b}^{2}}f(x)+3b{{\{f(x)\}}^{2}}\]\[-\{f{{(x)}^{3}}\}{{]}^{1/3}},\]for every\[x\]. Then,\[f(x)\]is a periodic function with the period

    A)
     \[a\]                

    B)
     \[2a\]

    C)
     \[b\]                

    D)
     \[2b\]

    View Answer play_arrow
  • question_answer251) If\[\overrightarrow{A}.\overrightarrow{B}=\overrightarrow{A}.\overrightarrow{C},\]then difference of\[\overrightarrow{B}\]and\[\overrightarrow{C}\]and\[\overrightarrow{A}\]are

    A)
     parallel          

    B)
     perpendicular

    C)
     coplanar         

    D)
     None of these

    View Answer play_arrow
  • question_answer252) If\[A=\left[ \begin{matrix}    \cos \alpha  & \sin \alpha   \\    -\sin \alpha  & \cos \alpha   \\ \end{matrix} \right],\]then\[{{A}^{2}}\]is equal to

    A)
     \[\left[ \begin{matrix}    \cos 2\alpha  & \sin 2\alpha   \\    -\sin 2\alpha  & \cos 2\alpha   \\ \end{matrix} \right]\]

    B)
    \[\left[ \begin{matrix}    {{\cos }^{2}}\alpha  & -{{\sin }^{2}}\alpha   \\    -{{\sin }^{2}}\alpha  & {{\cos }^{2}}\alpha   \\ \end{matrix} \right]\]

    C)
     \[\left[ \begin{matrix}    {{\sin }^{2}}\alpha  & {{\cos }^{2}}\alpha   \\    -{{\cos }^{2}}\alpha  & {{\sin }^{2}}\alpha   \\ \end{matrix} \right]\]

    D)
     \[\left[ \begin{matrix}    1 & 0  \\    0 & 1  \\ \end{matrix} \right]\]

    View Answer play_arrow
  • question_answer253) Argument of\[\frac{1+\sqrt{3}i}{\sqrt{3}+i}\]is

    A)
     \[\frac{\pi }{3}\]

    B)
     \[\frac{\pi }{2}\]

    C)
     \[0\]

    D)
     \[\frac{\pi }{6}\]

    View Answer play_arrow
  • question_answer254) The tangential point, where the line\[y=mx+c\]touches the parabola\[{{y}^{2}}=4ax,\]is

    A)
     \[\left( \frac{a}{{{m}^{2}}},\frac{-2a}{m} \right)\]

    B)
     \[\left( -\frac{a}{{{m}^{2}}},-\frac{2a}{m} \right)\]

    C)
     \[\left( -\frac{a}{{{m}^{2}}},-\frac{2a}{m} \right)\]

    D)
     \[\left( \frac{a}{{{m}^{2}}},\frac{2a}{m} \right)\]

    View Answer play_arrow
  • question_answer255) \[\frac{(\cos \alpha +i\sin \alpha )(\cos \beta +i\sin \beta )}{(\cos \gamma +i\sin \gamma )(\cos \delta +i\sin \delta )}\]is equal to

    A)
     \[\cos (\alpha +\beta -\gamma -\delta )-i\sin (\alpha +\beta -\gamma -\delta )\]

    B)
     \[\cos (\alpha +\beta -\gamma +\delta )+i\sin (\alpha +\beta -\gamma +\delta )\]

    C)
     \[\sin (\alpha +\beta -\gamma -\delta )+i\cos (\alpha +\beta -\gamma -\delta )\]

    D)
     \[\cos (\alpha +\beta -\gamma -\delta )+i\sin (\alpha +\beta -\gamma -\delta )\]

    View Answer play_arrow
  • question_answer256) \[sinh\text{ }3x\] is equal to

    A)
     \[4sin{{h}^{3}}x-3sinh\text{ }x\]

    B)
     \[3\text{ }sin{{h}^{3}}x+4\text{ }sinh\text{ }x\]

    C)
     \[3\text{ }sinh\text{ }x+4\text{ }sin{{h}^{3}}x\]

    D)
     \[3\text{ }sin{{h}^{3}}x-4\text{ }sinh\text{ }x\]

    View Answer play_arrow
  • question_answer257) If\[z=\frac{\sqrt{3}+i}{-2},\]then\[{{z}^{69}}\]is equal to

    A)
     1                

    B)
     2

    C)
     \[-i\]               

    D)
     \[-1\]

    View Answer play_arrow
  • question_answer258) If\[x+\frac{1}{x}=2\cos \theta ,\]then\[x\]is equal to

    A)
     \[cos\text{ }\theta +i\text{ }sin\text{ }\theta \]    

    B)
     \[cos\text{ }\theta -i\text{ }sin\text{ }\theta \]

    C)
     \[cos\text{ }\theta \pm i\text{ }sin\text{ }\theta \]    

    D)
     None of these

    View Answer play_arrow
  • question_answer259) Projection of a line on the axes are 3, 4, 5 respectively, then its length is

    A)
     12               

    B)
     50

    C)
     \[5\sqrt{2}\]             

    D)
     None of these

    View Answer play_arrow
  • question_answer260) If\[\overrightarrow{a}\]and\[\overrightarrow{b}\]are the adjacent sides of a rhombus, then

    A)
     \[\overrightarrow{a}.\overrightarrow{a}=\overrightarrow{b}.\overrightarrow{b}\]

    B)
     \[\overrightarrow{a}.\overrightarrow{b}=0\]

    C)
     \[\overrightarrow{a}\times \overrightarrow{b}=0\]

    D)
     \[\overrightarrow{a}.\overrightarrow{a}=0\]

    View Answer play_arrow
  • question_answer261) \[\overrightarrow{a}[\overrightarrow{a}\times (\overrightarrow{a}\times \overrightarrow{b})]\]is equal to

    A)
     \[(\overrightarrow{a}.\overrightarrow{a})(\overrightarrow{a}\times \overrightarrow{b})\]

    B)
     \[(\overrightarrow{a}.\overrightarrow{a})(\overrightarrow{b}\times \overrightarrow{a})\]

    C)
     \[(\overrightarrow{b}.\overrightarrow{b})(\overrightarrow{a}\times \overrightarrow{b})\]

    D)
     \[(\overrightarrow{b}.\overrightarrow{b})(\overrightarrow{b}\times \overrightarrow{a})\]

    View Answer play_arrow
  • question_answer262) \[\overrightarrow{A}.(\overrightarrow{B}\times \overrightarrow{C})\]is equal to

    A)
     \[\overrightarrow{B}.(\overrightarrow{A}\times \overrightarrow{C})\]

    B)
     \[\overrightarrow{B}.(\overrightarrow{C}\times \overrightarrow{A})\]

    C)
     \[\overrightarrow{C}.(\overrightarrow{B}\times \overrightarrow{A})\]

    D)
     None of these

    View Answer play_arrow
  • question_answer263) If\[O\]is the origin and\[{{P}_{3}}\]is the mid point of\[{{P}_{1}}(2,-1)\]and\[{{P}_{2}}(-4,3),\]then\[{{\overset{\to }{\mathop{OP}}\,}_{3}}\]is equal to

    A)
     \[(-1,1)\]

    B)
     \[(1,1)\]

    C)
     \[(1,-1)\]

    D)
     \[(-1,-1)\]

    View Answer play_arrow
  • question_answer264) \[\frac{\cos \sqrt{x}}{\sqrt{x}}\]is equal to

    A)
     \[\sin \sqrt{x}+c\]      

    B)
     \[2\sin \sqrt{x}+c\]

    C)
     \[-\sin \sqrt{x}+c\]      

    D)
     \[-2\sin \sqrt{x}+c\]

    View Answer play_arrow
  • question_answer265) Sum of the infinite terms of the series\[1+\frac{4}{5}+\frac{7}{{{5}^{2}}}+\frac{10}{{{5}^{3}}}+....\]is

    A)
     \[\frac{16}{35}\]

    B)
     \[\frac{11}{8}\]

    C)
     \[\frac{35}{16}\]

    D)
     \[\frac{8}{16}\]

    View Answer play_arrow
  • question_answer266) Radical axis of the circles \[3{{x}^{2}}+3{{y}^{2}}-7x+8y+11=0\]and \[{{x}^{2}}+{{y}^{2}}-3x-4y+5=0\]is

    A)
     \[x+10y=2\]      

    B)
     \[x+10y+2=0\]

    C)
     \[x+10y=8\]      

    D)
     \[x+10y+8=0\]

    View Answer play_arrow
  • question_answer267) \[\underset{x\to 0}{\mathop{\lim }}\,\frac{x({{5}^{x}}-1)}{1-\cos x}\]is equal to

    A)
     \[4\text{ }log\text{ }5\]           

    B)
     \[2\text{ }log\text{ }5\]

    C)
     \[\frac{1}{2}log\text{ }5\]            

    D)
     None of these

    View Answer play_arrow
  • question_answer268) If the line\[x+2y=1\]is the polar of the circle \[{{x}^{2}}+{{y}^{2}}=5,\]then pole is

    A)
     \[(5,10)\]            

    B)
     \[(5,5)\]

    C)
     \[(-5,10)\]           

    D)
     \[(-5,-10)\]

    View Answer play_arrow
  • question_answer269) \[\cos (\alpha +i\beta )\]is equal to

    A)
     \[cos\text{ }\alpha \text{ }coch\text{ }\beta -2\text{ }sin\text{ }\alpha \text{ }sinh\text{ }\beta \]

    B)
     \[cos\text{ }\alpha \text{ }cosh\text{ }\beta +2\text{ }sin\text{ }\alpha \text{ }sinh\text{ }\beta \]

    C)
     \[cosh\text{ }\alpha \text{ }cos\text{ }\beta -2\text{ }sinh\text{ }\alpha \text{ }sin\text{ }\beta \]

    D)
     None of the above

    View Answer play_arrow
  • question_answer270) If\[\overrightarrow{A}=2\hat{i}-2\hat{j}-\hat{k}\]and\[\overrightarrow{B}=-\hat{i}+3\hat{j}+2\hat{k},\]then \[A\times B\]is equal to

    A)
     \[\hat{i}+2\hat{j}-4\hat{k}\]     

    B)
     \[-\hat{i}-3\hat{j}+4\hat{k}\]

    C)
     \[-\hat{i}-5\hat{j}+4\hat{k}\]    

    D)
     \[-2\hat{i}-\hat{j}+\hat{k}\]

    View Answer play_arrow
  • question_answer271) If\[\overrightarrow{A}=4\hat{i}+8\hat{j}-\hat{k},\overrightarrow{B}=3\hat{i}+2\hat{j}-\hat{k}\]and\[\overrightarrow{C}=5\hat{i}+4\hat{j}+\hat{k},\]then\[\overrightarrow{A}.(\overrightarrow{B}.\overrightarrow{C})\]is equal to

    A)
     \[88\hat{i}+176\hat{j}-22\hat{k}\]

    B)
     \[22\hat{i}+44\hat{j}-11\hat{k}\]

    C)
     \[\hat{i}+4\hat{j}-\hat{k}\]

    D)
     None of these

    View Answer play_arrow
  • question_answer272) If A and B are two matrices such that\[AB=0,\]then

    A)
     \[A\ne O\]or\[B=O\]

    B)
     \[A=O\]and\[B=O\]

    C)
     it is not necessary that\[A=O\]or\[B=O\]

    D)
     all the statements are incorrect

    View Answer play_arrow
  • question_answer273) If\[\overrightarrow{a}\]and\[\overrightarrow{b}\]are non-collinear vectors and \[x\overrightarrow{a}+y\overrightarrow{b}=0,\]then

    A)
    \[x=0\]but it is not necessary for y to be 0

    B)
     not dependent on\[x\]and \[y\]

    C)
     \[x=0,y=0\]

    D)
     dependent on\[x\]and y

    View Answer play_arrow
  • question_answer274) A perpendicular to the pont P(1, 0,3) is drawn to the line joining the points A (4,7,1) and B(3,5,3). The foot of perpendicular to the line is

    A)
     \[(5,7,1)\]

    B)
     \[\left( \frac{5}{3},\frac{7}{3},\frac{17}{3} \right)\]

    C)
     \[\left( \frac{7}{3},\frac{5}{3},\frac{7}{3} \right)\]

    D)
     \[\left( \frac{5}{2},\frac{2}{3},\frac{7}{3} \right)\]

    View Answer play_arrow
  • question_answer275) If the coefficient of\[{{x}^{7}}\]and\[{{x}^{8}}\]in the expansion of\[{{\left( 2+\frac{x}{3} \right)}^{n}}\]are equal, then n is equal to

    A)
     55               

    B)
     56

    C)
     45              

    D)
     15

    View Answer play_arrow
  • question_answer276) Sum of the\[n\]terms of the series \[\frac{3}{{{1}^{2}}}+\frac{5}{{{1}^{2}}+{{2}^{2}}}+\frac{7}{{{1}^{2}}+{{2}^{2}}+{{3}^{2}}}+....\]is

    A)
     \[\frac{4n}{(n+1)}\]

    B)
     \[\frac{6n}{(n+1)}\]

    C)
     \[\frac{12n}{(n+1)}\]

    D)
     \[\frac{9n}{(n+1)}\]

    View Answer play_arrow
  • question_answer277) If difference of the roots of the equation \[{{x}^{2}}+px+8=0\]is 2, then value of p is

    A)
     \[\pm 6\]              

    B)
     \[\pm 2\]

    C)
     \[2,-6\]             

    D)
     \[6,2\]

    View Answer play_arrow
  • question_answer278) If the roots of the equation\[\frac{{{x}^{2}}-bx}{ax-c}=\frac{\lambda -1}{\lambda +1}\]are equal and opposite in sign, then\[\lambda \]is equal to

    A)
     \[\frac{a+b}{a-b}\]

    B)
     \[\frac{a-b}{a+b}\]

    C)
     0                

    D)
     None of these

    View Answer play_arrow
  • question_answer279) \[\int{\left( \frac{a{{x}^{3}}+b{{x}^{2}}+c}{{{x}^{4}}} \right)}dx\]is equal to

    A)
     \[a\log x+\frac{b}{{{x}^{3}}}+\frac{c}{3{{x}^{3}}}+A\]

    B)
     \[a\log x+\frac{b}{x}-\frac{c}{3{{x}^{3}}}+A\]

    C)
     \[a\log x-\frac{b}{x}-\frac{c}{3{{x}^{3}}}+A\]

    D)
     \[a\log x+\frac{b}{x}+\frac{c}{3{{x}^{3}}}+A\]

    View Answer play_arrow
  • question_answer280) \[\int{\sqrt{{{x}^{2}}+{{a}^{2}}}}dx\]is equal to

    A)
     \[\frac{x}{2}\sqrt{{{x}^{2}}+{{a}^{2}}}+\frac{{{a}^{2}}}{2}\log \{x+\sqrt{{{x}^{2}}+{{a}^{2}}}\}+c\]

    B)
     \[\frac{x}{2}\sqrt{{{x}^{2}}+{{a}^{2}}}-\frac{{{a}^{2}}}{2}\log \{x+\sqrt{{{x}^{2}}+{{a}^{2}}}\}+c\]

    C)
     \[\frac{x}{2}+\sqrt{{{x}^{2}}+{{a}^{2}}}+\frac{{{a}^{2}}}{2}\log \{x-\sqrt{{{x}^{2}}+{{a}^{2}}}\}+c\]

    D)
     \[\sqrt{{{x}^{2}}+{{a}^{2}}}+\frac{{{a}^{2}}}{2}\log \{x-\sqrt{{{x}^{2}}+{{a}^{2}}}\}+c\]

    View Answer play_arrow
  • question_answer281) If 6 points out of 10 points are collinear, then number of triangles made by using these 10 points is

    A)
     100              

    B)
     16

    C)
     12               

    D)
     14

    View Answer play_arrow
  • question_answer282) If\[{{a}_{ij}}=\frac{1}{2}2i-3j,\]then matrix\[{{A}_{2\times 2}}=[{{a}_{ij}}]\]is equal to

    A)
     \[\left[ \begin{matrix}    1/2 & 2  \\    1/2 & 1  \\ \end{matrix} \right]\]

    B)
     \[\left[ \begin{matrix}    2 & 2  \\    1/2 & 1/2  \\ \end{matrix} \right]\]

    C)
     \[\left[ \begin{matrix}    1/2 & -2  \\    -1/2 & 1  \\ \end{matrix} \right]\]

    D)
     None of these

    View Answer play_arrow
  • question_answer283) If\[A=\left[ \begin{matrix}    1 & -2 & 1  \\    2 & 1 & 3  \\ \end{matrix} \right]\]and\[B=\left[ \begin{matrix}    2 & 1  \\    3 & 2  \\    1 & 1  \\ \end{matrix} \right],\]then\[{{(AB)}^{T}}\]is equal to

    A)
     \[{{A}^{T}}{{B}^{T}}\]

    B)
     \[{{B}^{T}}{{A}^{T}}\]

    C)
     \[{{(AB)}^{-1}}\]

    D)
     \[A.B\]

    View Answer play_arrow
  • question_answer284) Probabilities to solve a problem by three students A, B and C are 1/2, 1/3 and 1/4, then the probability that the problem will be solved, is

    A)
     1/2

    B)
     1/4

    C)
     3/4

    D)
     2/3

    View Answer play_arrow
  • question_answer285) Two cards are drawn with replacement from a well shuffled pack of 52 cards. The probability that both are ace, is

    A)
     4/13             

    B)
     1/169

    C)
     2/13            

    D)
     None of these

    View Answer play_arrow
  • question_answer286) Length of normal at any point of the curve\[x=a(t+\sin t),y=a(1-\cos t),\]is

    A)
     \[a\sin t\]

    B)
    \[2a\sin {{t}^{3}}\left( \frac{t}{2} \right)\sec \left( \frac{t}{2} \right)\]

    C)
     \[2a\sin \left( \frac{t}{2} \right)\tan \left( \frac{t}{2} \right)\]

    D)
     \[2a\sin \left( \frac{t}{2} \right)\]

    View Answer play_arrow
  • question_answer287) \[\underset{x\to 0}{\mathop{\lim }}\,{{\left[ \tan \left( \frac{\pi }{4}+x \right) \right]}^{1/x}}\]is equal to

    A)
     \[{{e}^{2}}\]              

    B)
     e

    C)
     \[{{e}^{-1}}\]             

    D)
     1

    View Answer play_arrow
  • question_answer288) \[^{x}{{C}_{r}}{{+}^{x}}{{C}_{r-1}}^{y}{{C}_{1}}{{+}^{x}}{{C}_{r-2}}^{y}{{C}_{2}}+{{.....}^{y}}{{C}_{r}}\]is equal to

    A)
     \[\frac{x+y!}{r!}\]

    B)
     \[^{x+y}{{C}_{r}}^{xy}{{C}_{r}}\]

    C)
     \[^{x+y}{{C}_{r}}\]

    D)
     \[^{xy}{{C}_{r}}\]

    View Answer play_arrow
  • question_answer289) If \[A=\left[ \begin{matrix}    3 & 8  \\    2 & 1  \\ \end{matrix} \right],\]then\[{{A}^{-1}}\]is equal to

    A)
     \[-\frac{1}{13}\left[ \begin{matrix}    1 & 7  \\    8 & 3  \\ \end{matrix} \right]\]

    B)
     \[\left[ \begin{matrix}    -1 & 8  \\    2 & -3  \\ \end{matrix} \right]\]

    C)
     \[-\frac{1}{13}\left[ \begin{matrix}    1 & -8  \\    -2 & 3  \\ \end{matrix} \right]\]

    D)
     does not exist

    View Answer play_arrow
  • question_answer290) Direction ratios of two lines are\[5,-12,13\]and \[-3,4,5,\]then angle between the lines is

    A)
     \[{{\cos }^{-1}}\left( \frac{2}{65} \right)\]

    B)
     \[{{\cos }^{-1}}\left( \frac{1}{65} \right)\]

    C)
     \[{{\cos }^{-1}}\left( \frac{3}{65} \right)\]

    D)
     \[\frac{\pi }{3}\]

    View Answer play_arrow
  • question_answer291) In a GP, the ratio of the sum of first three terms and first six terms is 125:152, then common ratio is

    A)
     \[\frac{1}{5}\]

    B)
     \[\frac{2}{5}\]

    C)
     \[\frac{3}{5}\]

    D)
     \[\frac{4}{5}\]

    View Answer play_arrow
  • question_answer292) Triangle made by the points\[(0,7,10),(-1,6,6)\]and\[(-4,9,6)\]is

    A)
     right angled

    B)
     isosceles

    C)
     equilateral

    D)
     right angled isosceles

    View Answer play_arrow
  • question_answer293) If the equation\[a(b-c){{x}^{2}}+b(c-a)x+\]\[c(a-b)=0\]has equal roots, then a, b, c are in

    A)
     AP             

    B)
     HP

    C)
     GP             

    D)
     None of these

    View Answer play_arrow
  • question_answer294) If P=0.25, P=0.5 and\[P(A\cap B)=0.14,\]then\[P(\overline{A}\cap \overline{B})\]is equal to

    A)
     0.39             

    B)
     0.61

    C)
     0.48             

    D)
     0.45

    View Answer play_arrow
  • question_answer295) If\[y=\sqrt{\sin x+y},\]then\[\frac{dy}{dx}\]is equal to

    A)
     \[\frac{\cos x}{2y-1}\]

    B)
     \[\frac{\sin x}{2y-1}\]

    C)
     \[\frac{\log x}{1-2y}\]

    D)
     \[\frac{\sin x}{1+xy}\]

    View Answer play_arrow
  • question_answer296) Three vertices of a parallelogram are\[A(-1,-2),\]\[B(1,0)\]and\[C(3,4),\]then the fourth vertex is

    A)
     (1, 2)            

    B)
    \[(-3,-4)\]

    C)
     (2, 0)             

    D)
    \[(-1,2)\]

    View Answer play_arrow
  • question_answer297) If\[2x-\left[ \begin{matrix}    1 & 2  \\    7 & 4  \\ \end{matrix} \right]=\left[ \begin{matrix}    3 & 2  \\    0 & -2  \\ \end{matrix} \right],\]then\[x\]is equal to

    A)
     \[\left[ \begin{matrix}    4 & 4  \\    7 & 2  \\ \end{matrix} \right]\]

    B)
     \[\left[ \begin{matrix}    2 & 2  \\    7/2 & 1  \\ \end{matrix} \right]\]

    C)
     \[\left[ \begin{matrix}    3 & -1  \\    7/2 & 2  \\ \end{matrix} \right]\]

    D)
     None of these

    View Answer play_arrow
  • question_answer298) \[\int{\frac{{{x}^{2}}}{{{x}^{2}}+4}}\]is equal to

    A)
     \[x-4{{\tan }^{-1}}\left( \frac{x}{2} \right)+c\]

    B)
     \[x-2{{\tan }^{-1}}\left( \frac{x}{2} \right)+c\]

    C)
     \[x+4{{\tan }^{-1}}\left( \frac{x}{2} \right)+c\]

    D)
     None of the above

    View Answer play_arrow
  • question_answer299) The function \[f(x)=sin\left( \frac{\pi x}{2} \right)+2cos\left( \frac{\pi x}{3} \right)-\tan \left( \frac{\pi x}{4} \right)\]is a periodic function. It's period is

    A)
     6                 

    B)
     3

    C)
     4                 

    D)
     12

    View Answer play_arrow
  • question_answer300) Line\[ax+by+c=0\]is normal to the circle\[{{x}^{2}}+{{y}^{2}}={{r}^{2}}\]. Length of the intercepts cut by the line on the circle is

    A)
     r                 

    B)
     \[{{r}^{2}}\]

    C)
     2r               

    D)
     \[\sqrt{r}\]

    View Answer play_arrow

Study Package

   


You need to login to perform this action.
You will be redirected in 3 sec spinner